Sei sulla pagina 1di 61

Fanconi anemia- a type of idiopathic refractory anemia characterized by pancytopenia, hypoplasia of the bone marrow, and congenital anomalies,

occurring in members of the same family (an autosomal recessive trait); the anemia is normocytic or slightly macrocytic, macrocytes and target cells may be found in the circulating blood, and the leukopenia usually is due to neutropenia. Congenital anomalies include short stature; microcephaly; hypogenitalism; strabismus; anomalies of the thumbs, radii, and kidneys and urinary tract; mental retardation; and microphthalmia.

Fanconi syndrome is a disease of the pro imal renal tubules of the kidney in which glucose, amino acids, uric acid, phosphate and bicarbonate are passed into the urine, instead of being reabsorbed. !he loss of bicarbonate results in !ype " or pro imal renal tubular acidosis. !he loss of phosphate results in the bone disease rickets(even with ade#uate vitamin $ and calcium), because phosphate is necessary for bone development. %ilirubin occurs in plasma in four forms (&elson '()e Chapter-*+"), free or unbound bilirubin (the form responsible for kernicterus, because it can cross cell membranes) uncon-ugated bilirubin tightly bound to albumin

con-ugated bilirubin (the only fraction to appear in urine). .n the liver it is con-ugated with
glucuronic acid by the enzyme /lucuronyltransferase, making it soluble in water.

0 fraction- the fraction of bilirubin covalently bound to albumin1 in conventional


methods it is measured as part of con-ugated bilirubin. %ecause of its covalent bond during the recovery phase of hepatocellular -aundice, it may persist in the blood for a week or more after urine clears. 2lthough the terms direct and indirect bilirubin are used e#uivalently with con-ugated and uncon-ugated bilirubin, this is not #uantitatively correct, because the direct fraction includes both con-ugated bilirubin and 0 bilirubin.

Hepatorenal syndrome (345) is defined as functional renal failure in patients with end-stage liver disease. !he pathophysiology of 345 is poorly defined, but the hallmark is intense renal vasoconstriction (mediated by hemodynamic, humoral, or neurogenic mechanisms) with coe istent systemic vasodilation. !he diagnosis is supported by the findings of oliguria (6' m7)kg)day), a characteristic pattern of urine electrolyte abnormalities (urine sodium of 6'8 m9#)7, fractional e cretion of sodium of 6':, urine , plasma creatinine ratio 6'8, and normal urinary sediment), absence of hypovolemia, and e clusion of other kidney pathology. !he best treatment of 345 is timely liver transplantation, as complete renal recovery can be e pected. 9levations in serum 2;, +< nucleotidase (+<&!), and =-glutamyl transpeptidase (//!) levels are also sensitive indicators of obstruction or inflammation of the biliary tract. ;ancoast syndrome is a)w squamous cell ca (not adeno) Herpes simplex encephalitis (HSE) m)c viral encephalitis 1.n children older than * months and in adults, herpes simple encephalitis (359) is usually localized to the temporal and frontal lobes and is caused by herpes simple virus type ' (35>-'). .n neonates, however, brain involvement is generalized, and the usual cause is herpes simple virus type " (35>-"), which is ac#uired at the time of delivery.

359 must be distinguished from herpes simple meningitis, which is more commonly caused by 35>-" and which often occurs in association with a concurrent herpetic genital infection. C!- low density lesions in temporal lobes. 3emorrhage is highly suggestive and seen in later disease course. ?4.- temporal lobe hyperintense ?etavir 5coring is done for- Chronic heaptits &atalizumab in multiple sclerosis n Crohn@s as an antibody against alpha A integrin inhibiting ! lymphocyte adhesion n migration to lesions in these diseases. &9B classification liver cancer --CDEF$2, C7.; primary cause of community ac#uired pneumonia ----C ?452 albright@s osteodystrophy is &D4?27 calcium, not decreased calcium Plummer vinson's syndrome - Combination of symptomatic hypopharyngeal webs and .ron deficiency anemia in middle aged female. ;resence of web is in postcricoid1 region. 3emoglobin is low and there is hypochromic microcytic anemia. 2ll cases have a low serum iron and high iron binding capacity. .ron shold be given orally to correct 3b levels. 2trial fibrillation- 9nlarged left atrium and 1increased left atrial pressure. ;rocollagen suicide phenomenon seen in- D. (Dsteogenesis .mperfecta) !he etiologic agent in over G8: cases of post-transfusion hepatitis is hepatitis C. .t is also associated with incidence of chronic active hepatitis upto 'H: and an 1(-'8: increase of cirrhosis or hepatoma or both. C?> infection is not a significant problem in immunocompetent recipients and hence blood is not routinely tested for C?>. (I-'J( /!-(J) Diabetes insipidus(DI) is a condition characterized by e cessive thirst and e cretion of large amounts of severely diluted urine, with reduction of fluid intake having no effect on the latter. !here are several different types of $., each with a different cause. !he most common type in humans is central $., caused by a deficiency of arginine vasopressin(2>;), also known as antidiuretic hormone(2$3). >asopressin acts at the distal tubule and collecting ducts to increase permeability to water not a)w electrolytes i.e. free water. 3ence central $. is a)w 1free waterclearence. Fnder normal circumstances, this would result in the movement of free water from renal tubule to capillary. Fnder conditions of vasopressin deficiency, as in nephrogenic $. water remains in the tubules, resulting in e cretion of dilute urine. !he second common type of $. is nephrogenic diabetes insipidus, which is caused by an insensitivity of the kidneys to 2$3. 5ymptoms are similar to neurogenic $. e cept plasma levels of vasopressin are increased. Hyperuricemia- 2 purine rich diet can cause hyperuricemia. Koods high in the purines adenine and hypo anthine may be more potent in e acerbating hyperuricemia. 7ow temperature is a commonly reported trigger of acute gout. !his is believed to be due to temperature dependent precipitation of uric acid crystals in tissues at below normal temperature. !hough asymptomatic hyperuricemia does increase the risk of acute gouty arthritis prophylla is is not recommended. 3%e2g negative means Chronic carrier low infectivity ;recore mutant (infective but still 3%e2g negative) both can be differentiated by >.427 7D2$ , 27! 79>97 Fanconi anemia- a type of idiopathic refractory anemia characterized by pancytopenia, hypoplasia of the bone marrow , and congenital anomalies. !he anemia is normocytic or slightly macrocytic, macrocytes and target cells may be found in the circulating blood, and the leukopenia usually is due to neutropenia. Congenital anomalies include short stature; microcephaly; hypogenitalism; strabismus; anomalies of the thumbs, radii, and kidneys and urinary tract; mental retardation; and microphthalmia on!enital hypoplastic anemia- hypoproliferative macrocytic anemia. 2 macrocytic anemia resulting from congenital hypoplasia of the bone marrow, which is grossly deficient in erythroid precursors while other elements are normal; anemia is progressive and severe, but leukocyte and platelet counts are normal or slightly reduced; survival of transfused erythrocytes is normal; minor congenital anomalies are found in some patients. (5yn, congenital nonregenerative anemia, Diamond-Blackfan anemia , familial hypoplastic anemia, pure red cell anemia, erythrogenesis imperfecta, Diamond-Blackfan syndrome .)

"- #hich o$ the $ollo%in! is not an indication o$ Digoxin-specific Fab antibodies in children&' Supraventricular tachycardia------ans (' )verdose * + m! ,' Serum di!oxin levels * &-n!.ml +' Pro!ressive bradycardia 9 planation!reatment of $igo in to icity 5upraventricular tachycardia- ECl Kre#uent >entricular premature beats- 7idocaine);henytoin >entricular tachycardia)Kibrillation- Cardioversion)$igibind 5econd or third degree heart block- 2tropine)Catheter);acing)$igibind .ndications of digibind are'. 2ccidental overdose CAmg ". 5erum digo in level C '8 ng)ml *. 7ife threatening rhythm disturbance>entricular arrhythmias);rogressive bradycardia("nd or *rd degree heart block) "' / +- yr old male alcoholic presents %ith a 0 days history o$ bin!e drin1in!' Serum chemistry tests reveal the $ollo%in!2 Electrolytes- 3a4 &+56 74 56 l- &-56 H ),- &5 reatinine- &'5 m!.dl 8lucose- &9( :he nitroprusside a!ent !ives a minimally positive result' )ptimal therapy to ameliorate the patient's acid-base disorder %ould include 5; dextrose in'. Bater ". &5-------------------------------------------------ans *. &5, insulin, and sodium bicarbonate A. ')" &5 and insulin $iscussion- 3)'J ;-"G')"G" ?etabolic acidosis d)t alcohol (alcoholic ketoacidosis). ;atients suffering from alcoholic ketoacidosis do well on glucose and saline. &either insulin nor alkali is re#uired unless the acidosis is e treme (bicarbonate 6H to ()

"' /ll are tests $or stoppa!e o$ circulation E< EP:2 '. ?agnus test ". Binslow@s test------------ans *. .card@s test A. $iaphanous test $iscussionBinslowLs test- &o movement of reflection of light shown on mirror, or surface of water in bowl, kept on the chest. .ts a !est of respiration.

!he se#uence of events in 2.;(" nd ?)C porphyryia) attacks usually is 2bdominal pain ;sychiatric symptoms, such as hysteria ;eripheral neuropathies, mainly motor neuropathies ?ost patients are completely free of symptoms between attacks 2.; displays neurovisceral symptoms but no skin manifestations.

!he 5+ heart sound is associated with any process that increases the stiffness of the ventricle including,

hypertrophy of the ventricle long-standing hypertension (causes ventricular hypertrophy) aortic stenosis (causes ventricular hypertrophy)
overloading of the ventricle (causes ventricular hypertrophy)

fibrosis of the ventricle (eg. post-?.)


Congestive 3eart Kailure 7ymphoma of the brain (ususally diffuse large cell) is increasingly common as a sporadic tumor and occurs fre#uently in immunosuppressed patients, specially those with 2.$5. .t@s clinical sensitivity to glucocorticoids can mistakenly suggest a diagnosis of multiple sclerosis, and its complete disappearence or dramatic improvement on C! after steroid therapy is baffling. 4adiosensitivity is a well known feature of most primary C&5 lymphomas, which almost always are of 1%-Cell origin. Fibromyal!ia- 2 history of sleep distrbances and widespread musculoskeletal pain associated with trigger points1 (localised area of tenderness on palpation). /limentary !lycosuria , /lycosuria developing after the ingestion of a moderate amount of sugar or starch, which normally is metabolized without appearing in the urine. 5een in hyperthyroidism. .n 1-alpha-hydroxylase deficiency deficiency of >it. $ can not be corrected by supplementation. /uerbach's re!ime used in treatment of--C ?alignant pheochromocytoma Sarcoidosis- mostly incidious but presents acutely as 3eerfordt Baldenstrom syndrome 1- fever, parotid enlargement, anterior uveitis, and facial nerve palsy 7ofgren@s syndrome - erythema nodosum, arthralgias, and bilateral hilar lymphadenopathy 2 normal ?C> within " days after acute bleeding--C because %? has not got enough time to release reticulocytes(which are larger than mature 4%Cs and could potentially increase the ?C>)

8onococcal urethritis

3on-speci$ic urethritis

/onococcal urethritis presents 1within the first &on-specific urethritis is most commonly caused by week of se ual e posure./onococccal Chlamydia trachomatis. !he .; is "-*wks following urethritis has a thick creamy e udate se ual e posure. ;urulent e udate in non-specific urethritis is thin and watery ;atient complain of dysuria and increased fre#uency of urination in both types of urethritis. /ram stain of e udate in both types has numerous nutrophils, but in gonoccocal urethritis, gram negative diplococci are visible in the cytoplasm. Chlamydia is not visible on routine gram stains Bilson@s d)e--C wrong statement--C Ceruloplasmin6'88 $efective copper incorporation into apoceruloplasmin leads to e cess catabolism and low blood levels of ceruloplasmin. Kew important points Chronic fundal gastritis a)w pernicious anemia Chronic antral gastritis a)w 3.pylori infection %arretts esophagus l)t adenocarcinoma 2chalasia cardia l)t s#uamous cell carcinoma 2llelic hetrogenicity is the phenomenon in which different mutations at the same locus(or gene) cause the same disorder. 9g. %eta-thalassemia

"' / lady on ri$ampicin = %ar$arin develop pulmonary embolism' :he best mana!ement is')4eplace warferin by long term -eparin therapy ")/ive 7?B3 *)4eplace warferin A acetacumarin A)4eplace rifampicin A ethambutol-----------ans given $iscussion') 5ince the lady is on rifampicin she must b suffering frm tb or leprosy...2s rifampicin is cidal drug and most imp drug in treatmemt regimen of both tb and leprosy, it cannot be replaced by ethambutol, which is a satic drug. 5o i

think warfarin should be stopped and replaced by 7?B3. ")Mea, u r right. %ut we have no such option in the #uestion. !he second option says that N/ive 7?B3N,,,, not Nreplace warfarin with 7?B3N. 4ifampin do induce the metabolism of Barfarin in liver and affects .&4. !he induction of metabolism is unpredictable. 7ong term heparin therapy as such carries a lot of risk of complications. 2cenocumarin also interact with 4ifampin. 5o, the Ath Dption is the best option among all of these. .soniazid is also a -Cidal drug, so shd be there in therapy too. "- Di!italis act in atrial $ibrillation by')increasing 2> node refractoriness ")decreasing atrial contractility *)inhibiting &aOEO2!;ase A)inhibiting &aO3O2!;ase $iscussion- 2ns is -') %asic mechanism to &aOEO2!;ase inhibition h,then why the ans. .s ')... $igitalis also acts on the >agus nerve through the same channel, and increases its discharge(i.e., it increases vagus tone). .ncreased vagus tone in turn inhibits 2> node and make it refractory.5o, it prevents the impulses generated in atria to reach >entricles, control ventricular rate and improves hemodynamic stability in 2trial fibrillation. "' Dama!e to cate!orial hemisphere usually leads toa.normal speech b.increased speech c.decreased speech answer d.senseless fluent speech $iscussion- ans given for this in aa is PdQ also causes $ysle ia which is a broad term applied to impaired ability to read, due to an inherited abnormality. Causes of $ysle ia, 4educed ability to recall speech sounds, so there is trouble translating them mentally into sound units (phonemes). !here is a defect in the magnocellular portion of the visual system that slows processing and also leads to phonemic deficit. !here is decreased blood flow in angular gyrus in categorical hemisphere in both cases. "' >ost sensitive dia!nostic test + den!ue isa..g? elisa b.complement fi ation test c.neutrilization test answer... routinely done at airports in china n taiwan d.electron microscopy $iscussion- Kamous by name nsst...... igm elisa was usedd to b... now it is nsst G+: sp n GG: sn...

Primary Immunode$iciency disorders ,

;rimary ?-cell diseases include panhypogammaglobulinemia (?ruton disease), an R-linked


deficiency of all three ma-or classes of immunoglobulins, as well as other selective deficiencies of the immunoglobulins or their subgroups. !his condition presents after * months of age (after maternal antibodies wane) with recurrent and often simultaneous bouts of otitis media, pneumonia, diarrhea, and sinusitis.

:-cell, 2mong the !-cell diseases is $i/eorge anomaly, in which defective embryologic
development of the third and fourth pharyngeal pouches results in hypoplasia of both thymus and parathyroid glands. 2ssociated findings with $i/eorge anomaly include C2!C3, C for cardiac, 2 for abnormal faces, ! for thymic hypoplasia, C for cleft palate, and 3 for hypocalcemia.

ombined ? and :-cell diseases,

R-linked recessive Biskott-2ldrich syndrome of mild !-cell dysfunction, diminished


serum .g?, marked elevation of .g2 and .g9, eczema, recurrent middle-ear infections, lymphopenia, and thrombocytopenia.

5evere combined immunodeficiency disease (5C.$), have deficient !- and %-cells.


Conse#uently, they are both marked lymphopenia and agammaglobulinemia, as well as hypoplasia of the thymus.

2ta ia telangiectasia and chronic mucocutaneous candidiasis.


Complication of malaria (C32;7.&), Cerebral malaria) Coma 3ypoglycemia 2naemia ;ulmonary edema 7actic acidosis .nfections &ecrois of renal tubules (2!&) wind swept deformity of hands------rheumatoid arthritis wind swept deformity of knee------rickets most common cause of pyometra in india---carcinoma cervi in western(developed)countries-----senile endometritis

when nothing mentioned ,,we should choose whats commoner in india and hence choose cancer cervi as the answer.

.n )PD,

?ronchitis has ?... so they are called N?lue bloatersN. Cyanosis is characteristic finding. emPhysema has P...so they are called NPink ;uffersN.
?ehcet's syndrome '. 4ecurrent genital ulcers ". 5kin lesions *. 9ye lesions A. ;athergy test

Supraventricular tachycardia, treatment 2%C$9,

2denosine %eta-blocker Calcium channel antagonist $igo in 9 citation (vagal stimulation)


@entricular tachycardia, treatment 72?%,

7idocaine

2miodarone ?e iltene) ?agnesium %eta-blocker


&eurofibromatosis, diagnositic criteria (type-') C2K9 5;D!, -Cafe-au-lait spots -2 illary-.nguinal -Kreckling Kibroma -9ye, lisch nodules -5keletal (bowing leg, etc) -;edigree) ;ositive family history -Dptic !umor (glioma)

"' #hich o$ the $ollo%in! is true re!ardin! /sthma2) 4educed K4C, 4educed 4esidual >olume %) 4educed K4C, .ncreased 4esidual >olume C) .ncreased K4C, 4educed 4esidual >olume $) .ncreased K4C, .ncreased 4esidual >olume-------------ans &ecrobiosis lipoidica diabeticorum--C found over anterior surface of legs ;ersons with ;> usually have diminished cerebral blood flow and are particularly at risk for developing thrombotic complications. Kunctional platelet abnormalities may cause both thrombotic and bleeding problems(gi tract is common site of bleeding), and affected persons are fre#uently iron-deficient. ! wave inversion of the anterior leads especially >'->A seen in--Cpulmonary embolism. /eneva score is used in pul. embolism

/nti mitochondrial antibody Primary biliary cirrhosis 2nti smooth muscle antibody 2utoimmune hepatitis

Coma vigil--Cakinetic mutism--Ca state in which patient lies with eyes open, yet unresponsive to the outside world--Cseen in typhoid >ycoplasma pneumoniae--Cdiagnosis is confirmed by Cold agglutinins. 2s in other atypical pneumonias, radiographic abnormalities may be more prominant than would be predicted by auscultation of chest (normal or nearly normal) 7issencephaly--Csmooth brain (lack development of gyri and sulci)

"- ?reath sounds are decreased in all except'. 7obar pneumonia-------ans ". ;leural effusion *. ;neumothora A. 2telectasis 9 planation- $ue to consolidation in lobar pneumonia, high pitched bronchial breadth sound and whispering pectorilo#uy are present "'#hich o$ the $ollo%in! is true re!ardin! Aheumatoid arthritis - (/l B+) a) !ypically involves small and large -oints symmetrically but spares the cervical spine b) Causes pleural effusion with low sugar c) ;ulmonary nodules are absent d) 9nthesopathy prominent

$iscussion- /3S I3 //,2ns. is @a@ i.e., !ypically involves small and large -oints symmetrically but spares the cervical spine CD449C! 2&5 %--C 3244.5D& 'J,"8(J--C ;leuropulmonary manifestations, which are more commonly observed in men, include pleural disease, interstitial fibrosis, pleuropulmonary nodules, pneumonitis, and arteritis. 9vidence of pleuritis is found commonly at autopsy, but symptomatic disease during life is infre#uent. !ypically, the pleural fluid contains very low levels of glucose in the absence of infection. ;leural fluid complement is also low compared with the serum level when these are related to the total protein concentration "' Hyperbaric oxy!en is use$ul in the treatment o$ all6 except2 '. CD poisoning ". /as gangrene *. 2telectasis-----------------ans A. Cyanide poisoning 9 planation- .n atelectasis alveolar collapse leads to shunting of the desaturated blood at the alveolar capillary level and thus the desaturated blood is not available for o ygenation. ;lasma levels of many polypeptide hormones(parathyroid hormone, insulin, glucagons, 73 and prolactin) rise with renal failure. Fremic patients are also resistant to the action of insulin. !he glucose intolerance of uremia results mainly from this peripheral resistance to the action of insulin. Fanconi Syndrome is characterised by glycosuria, aminoaciduria, and phosphaturia(mnemonic 8/P). 2minoaciduria is generalised and other associated abnormalities may be bicarbonaturia, hyperkaluria, uricosuria, and sodium wasting. Clinical features are linear growth failure and vitamin $ deficiency rickets. 5ymptoms typically appear in 'st si months of life and consists of vomiting, polydipsia, polyuria and constipation. >ycosis $un!oides--C Cutaneous !-Cell lymphoma(&37)--C name mycosis fungoides is somewhat misleading--it loosely means Nmushroom-like fungal diseaseN $uring immediate *8 min postictal period serum prolactin level rises. !his does not happen in peudoseizure--C hence used to differentiate the two 2sthma may be aggravated by Chlamydia or mycoplasma pneumoniae. !reatment with macrolide antibiotics benefits patients with %2. .n asthma airway obstruction is reversible, ?ultiple sclerosis also causes facial palsy but taste is not lost unlike in bellLs palsy 2 lesion to the left prefrontal area produces depression and uncontrollable crying. .n contrast a comparable lesion to the right prefrontal area may produce laughter, euphoria and a tendency to -oke and make puns. M decent and 5* are produced in the rapid filling phase of cardiac cycle. !hatLs why 5* is also known as filling sound. ?ineral deficiencyo Cinc de$iciency is characterised by alopecia, a maculopapular rash around the mouth and eyes, taste dysguesia and smell abnormalities and problems with healing of wounds. o Essential $atty acid de$iciency is characterised by an eczematous rash and thrombocytopenia. o >a!nesium de$iciency is associated with resistence to activity of parathormone with subse#uent hypocalcemia and tetany. ?uscle weakness, irritability, delirium and convulsions are also noted. 2lcoholism is the most common cause of hypomagnesemia o opper de$iciency is associated with iron deficiency anemia, dissecting aortic aneurysm and kinky hair syndrome %ecause helper ! cells are integral to normal cellular immunity(type .> hypersensitivity) tests that evaluate cellular immunity are impaired in 3.>. !his lack of immune response is called anarchy. .n vitro stimulation of !-cell response to phytohemagglutinin, a potent !-cell mitogen is also impaired. DI - 2ntithrombin ... levels may be low due to the combination of increased consumption and decreased synthesis. Dther findings- 3ypofibrinogenemia, !hrombocytopenia, Kibrin degradation products and prolon!ed prothrombin time DhermitteEs si!n, also known as barber chair sign is an electrical shocklike sensation (induced by fle ion or other movements of neck) that radiates down the back into the legs. 4arely it radiates into the arms. Kound in >S, Cervical spondylosis S posterior column lesion. D@:-4isk factors, o ;!3(Tmnemonic) ;&3, ;olycythemia

o o

!hrombophilia--C activated protein C resistance, protein C and 5 deficiency 3ematological, 3yperviscosity syndrome, Homocysteinuria 2utoimmune-2ntiphospholipid syndrome $rugs 2ntipsychotics(recent evidence) DC;, *rd/ C "nd/

Pierre Aobin Syndrome - isolated cleft palate, retrognathia, and a posteriorly displaced tongue(glossitis) and is associated with early respiratory and feeding difficulties F-curve phenomenon- point beyond which blood pressure reduction in hypertensive sub-ects is no longer beneficial and possibly even deleterious. Clinical trials have found no evidence of U--curve phenomenonL at blood pressure reductions achieved in clinical practice. Channelopathy of Sodium, potassium and calcium are autosomal dominant linical Aelapse- .n type / hepatitis once the patient is fully cured he may again develop symptoms of fever, malaise, anore ia and vomiting without -aundice. !he 7K! is normal.

"' / +- yrs old lady under%ent cholecystectomy $or !all stones' (+ hrs a$ter the sur!ery she became con$used and developed tachycardia' )n examination lun!s %ere clear' Her arterial blood !as analysis sho%ed p)( G 5-6 p )(G (H6 pHG9'+B' :he most important next investi!ation to establish the cause o$ her deterioratin! condition %ould be'. Chest roentgenogram ". %lood culture *. 9C/ A. 7ung scan------------ans $iscussion- $evelopment of repiratory distress, hypo ia and confusion following surgery; without any obvious reason should make one to suspect P:E. Dther risk factors for ;!9 are Obesity, Pregnancy, OCPs etc. Dun! scannin! is the principal imaging test for diagnosis of ;!9, which detects perfusion defect indicating decreased or absent pulmonary blood flow. Chest R-4ay is normal or near normal in most cases of ;!9 and a normal or near normal chest R-4ay in a dyspoenic patient suggests ;9. Bell established abnormalities include focal oligemia( #estermar1Es si!n), a peripheral wedged shaped opacity above the diaphragm(1 HomptonEs hump), or an enlarged right descending pulmonary artery(PallaEs si!n). !he 9C/ is often abnormal in ;9, but findings are not sensitive, not specific. 9C/ changes include sinus tachycardia1, new onset atrial fibrillation or flutter and 5'I*!* (it is a sign of acute cor pulmonale ). 2ny cause of acute cor pulmonale (;9, ;!R, bronchospasm, etc) can result in the 5'I*!* finding on the electrocardiogram. /nterior : %ave inversions suggest the diagnosis of massive or sub-massive ;9. Overall the greatest utility of the EC in the patient with suspected PE is ruling out other potential life-threatening diagnoses such as !"# ;ericarditis in renal failure(acute or chronic) is an indication to initiate haemodialysis, because untreated uremic pericarditis may progress to pericardial temponade . Dther indications for haemodialysis include encephalopathy, volume overload, and intractable hyperkalemia. 2lso %? depression, mainly due to reduced erythropoietin combined with mildly reduced red cell half-life, causes hematocrit to fall almost universally in renal failure(acute and chronic). !his does not determine need for dialysis.

Causes of %ide pulse pressure, o 24 o 3yperthyroidism o ;regnancy o 5evere anemia o ;agetLs disease o %eri-beri !hese are also found in (are causes of), Bater hammer pulse ) 5ystolic 3!
)nset ourse onsciousness Dementia(??59 6"A) .nsidious Fsually protracted & Delirium 2cute Fsually recover in ' week Clouded

)rientation >emory omprehension Sleep-%a1e cycle /ttention and concentration Diurnal variation

& .mmediate retention and recall normal 4ecent memory disturbed 4emote memory disturbed only in late stages .mpaired only in late stages & & -nt

$isturbed .mmediate retention and recall disturbed1 4ecent memory disturbed .mpaired $isturbed $isturbed ?arked, Sundo%nin! 4nt--* Dnset or

Perception

3allucinations may occur

e acerbation of delirium during the evening or night >isual illusions and hallucinations very common

Kirst sign seen in hypoglycemia--C loss of fine motor skills #ernic1e's encephalopathy- classic triad(/D2)o /lobal confusion o Dphthalmoplegia o 2ta ia Bhen the diagnosis is suspected thiamine should be administered before glucose since glucose can precipitate worsening of the disease. ?any of the patients who recover from the acute encephalopathy will be left with a profound defect in memory and learning k)a 7orsa1o$$'s psychosis. $rugs that can be cleared with 3emodialysis- ?D/S: %arbiturate 7ithium 2lcohol(includes--C methanol, ethylene glycol) 5alicylates !heophylline(charcoal hemoperfusion preferred) 2cid diuresis- , "ueens P/ : Iuinine Iuinidine ChloroIuine ;hencyclidene 2mphetamine Cocaine !C2, !ocainide 2lkaline diuresis- >S PDF ?t 5alicylate, 5ulfonamide Chlorpropamide ;henobarbitone $iflunisal Kluorine 5aline diuresis- /? FID> 2lcohol %r Ca Kluorine .&3 7i ?eprobamate 3emoperfusion- ?E8 P>: %arbiturates 9thchlorvynol /lutethimide

Chloramphenicol ;henytoin);rocainamide ?eprobamate)?etha#ualone !heophylline $rugs which can-not be cleared with hemodialysis- ABCDs %nti-cholinergic&'C% ( Organophosphates) *en+odia+epines, *eta blockers Compound like kerosene oil $extropropoxyphene&Co-proxamol) $igoxin

!he psycho!enic stupor can be differentiated from organic stupor byo !he dollLs head eye phenomenon or oculocephalic refle --C absent in psychogenic stupor o Dculovestibular refle --C present o ;rotective refle es--C present o 4esistence to eye opening--C present

%ruit is found in hepatoma--C highly vascular tumor 4elative polycythemia (due to relative decrease in plasma volume)--C seen in den!ue hemorrha!ic $ever >it. 2 causes rupture of lysosomal membranes (but carotenoids do not cause to icity e cept a reversible yellow discolouration of skin)

/ADI)D)8I
Hypertension
&' >r' Sharma +0 years old has ?P o$ &5H.&-( he has'. &ormal %; ". ;re hypertensive *. 5tage . 3! A. 5tage .. 3!---------------ans $iscussion- &ew classification of 3ypertension(V&C >..), lassi$ication &D4?27 ;re-hypertension Sta!e & 5tage " "solated systolic ,' o o Systolic 6'"8 '"8-'*G &+--&5B C 'H8 - 1./ and and)or and.or and)or and Diastolic 6 (8 (8-(G B--BB C '88 0 1/

Cuff width should be *(.,J of arm circumference $iastolic pressure, disappearance of sounds (Eorotkoff >1)--C 2.;/9 "88( ;seudohypertension- ;seudohypertension is when blood pressure measurements are elevated but the blood pressure is actually normal. 5een in elderly d)t arterioloscerosis(not atherosclerosis)

(' 8oldblatt hypertension is seen in'. 4enovascular disease----------------ans ". $rug induced *. 2drenal medulla tumor A. Children $iscussion4enovascular disease (4enal artery stenosis)--C 8old ?latt 1idney, ?ost fre#uently atheromatous(elderly, cigarette smokers with peripheral vascular disease) or $ibro muscular dysphasiaJ in young ladies !he kidney with 425 becomes 1small and shrunken. !he decrease in bloodflow to the kidney with 425 (/oldblatt@s kidney) causes hyperplasia of V/ appratus and increased renin production

which lead to retention of sodium and also produces 3! but escapes the effects of 3! d)t stenosis. !he other kidney however shows microscopic shanges of benign nephrosclerosis (hyaline arteriolosclerosis) d)t effects of 3!. 425, can occur in :a1ayasu disease but it does not occur in ;2&

3ote, %est screening test for 425 is /adolium enhanced ?4. angiography.(&.I);reviously it used to be 34C!. ?ost con$irmatory test for 425 is conventional angiography.(K2I) Krom copy.ncreased renin'. 4. 4enal 2rtery ". 4. 4enal >ein-------------ans *. 7. 4enal 2rtery A. 7. 4enal >ein ,' /ll are causes o$ hypertension %ith hypo1alemia except'. %ilateral 425 ". C4K-----------------------ans(hyperkalemia) *. Conn syndrome A. Crushing syndrome $iscussion3! also occurs in 24K)2c. /lomerulonephritis ?)C cause of sec. 3!--C 4enal "' ?P in pheochromocytoma'. 5ustained 3! ". 5ustained 3! with post. hypo *. 9pisodic 3! A. 9pisodic 3! with post. hypo---------------ans(post.--C ;ostural Ck) $iscussion- (;T""HG 3)'J) K&2C not done in pheochromocytoma because--C 3ighly vascular ;heochromocytoma--C 5 --C %; decreases--C give &5 Hypertension >ana!ement2 ?P G ardiac output( )) x Peripheral resistance(PA) ) G Heart rate(HA) x Stro1e volume(S@) 5o, ?P G HA x S@ x PA

2. Dru!s %hich reduce HA- %eta blockers %. Dru!s %hich reduce stro1e volume'. ?eta bloc1ers2 beta blockers have negative inotropic S negative chronotropic effects (so beta blocker should be used with caution or should be avoided in HFJ with 3!) ". Diuretics, !hey reduce blood volume so they reduce the preload1 *. 3itrate,!hey primarily dilate the venules thereby cause peripheral pooling of blood. 5o they reduce the preload. C. Dru!s %hich reduce the peripheral resistance1# %lpha blockers 2# Calcium channel blockers 3# %CE-"4 A. $irect vasodilators W3ydralazine, alpha-?ethyl $opa, 5odium &itroprusside(2O>), indapamideX +. %osentan (&ew I)--C 9ndothelial receptor blocking drug (;ri. ;ul. 3!, 4aynaud@s d)e) &ew drugs for 3!- 2liskiren, Kendolpam 3ever use sublin!ual ni$edipineJ to reduce ?P (bi! drop in ?P and increase stro1e ris1 )(3") .n-ection frusemide should not be used in severe 3! %ut can be used in severe 3! in 7>K.(&I) %; T CD ;4 --C

'. ". *. A.

5ystolic $iastolic ?2;----------------ans ;; 9nd diastolic volume--C '"8 ml 9nd systolic volume--C A8 ml 5troke volume--C H- mlJ 9K--C ")* (HH:) &9K--C H+-J": (Ck) 5ingle best test to calculate 9K--C 9chocardiography

$iuretic not used in which 3! ;heochromocytoma (already dehydrated) also ;olycythemia(3)'J ;-*H")

HF
+' Earliest si!n o$ D@F'. 5* ". %asal crepitation *. !achycardia-------------------ans A. 4aise V>; $iscussion-(3)'J ;-'AA*) 7>K. of choice--C 9K '. 4esting tachycardia1 (9arliest feature of 7>K--C !achycardia) ". ;ulsus alternans1 *. 2uscultation, S, !allop1 (%est feature of 7>K but heard normally in Children, ;regnancy) 5igns of 4>K, Aaised F@P, 9dema 2scites , 3epatomegaly &5' Pulsus alternans is seen in'. 2ortic regurgitation ". 3ypertrophic cardiomyopathy *. 2-> block A. 5evere 7>K----------------ans $.5CF55.D&5 ?is$eriens pulse (!wo systolic peaks)- 241, 24 with 251, 3DC?1 (%isferiens pulse is best assessed in 1branchial.radial artery) Dicrotic pulse (!wo waves, Dne is systole and one is diastole)- $ilated cardiomyopathy4 . Pulsus alternans(regular alteration of amplitude with regular rhythm)- 5evere 7>K1. (5-* is found with ;ulsus alternans) Pulsus paradoxus (decrease in systolic arterial pressure during inspiration)- ;ericardial tamponade1, 5evere CD;$, %ronchial 2sthma1, 5>C obstruction 5' #hich is not a maKor Framin!hamEs criteria in HF'. Cardiomegaly ". ;aro ysmal nocturnal dyspnea *. 5-* gallop A. 3epatomegaly-----------------ans $iscussionFramin!ham criteria $or dia!nosis o$ HF , ?a-or criteria'. ;aro ysmal nocturnal dyspnea

". *. A. +. H. J. (.

4aised V>; Crepitations Cardiomegaly 2cute pulmonary edema 5-* .ncreased venous pressure(C'Hcm 3"D) ;ositive hepato-ugular reflu

?inor criteria'. 9 tremely edema ". Drthopnea(night cough) *. $yspnea on e cretion .# ,epatomegaly +. ;leural effusion H. >ital capacity reduced by ')* from normal J. !achycardia (C'"8)m)--C Earliest criteria even though a minor criteria 2t least one ma-or and two minor criteria are re#uired1 0' #hich o$ the $ollo%in! is not seen in HF'. .ncrease serum sodium--------------------------ans(2.;/ "88J) ". .ncrease catecolamines *. .ncrease blood urea A. ;edal edema $iscussionompensatory mechanism'. .ncrease epinephrine , .t leads to tachycardia ". .ncrease nor-epinephrine , .t leads to increase peripheral resistance, there-by blood supply to muscles and skin is reduced but blood supply to heart and brain is normal *. 4educe renal blood flow, .t lead to prerenal azotemia and hyperaldosteronism A. .ncrease aldosterone, .t leads to more &a and water retention but primary &aO gain is e ceeded by secondary water gain....(5ee ?E >ol-' ;-"G8 for more) ;ercentage of blood flow to Eidney- "8: C3K, (Cold e temities--C 7ess blood to skeletal muscle and skin) C3K, decreased bood flow to kidney--C hyperaldosteronism but dilutional hyponatremia 3yponatremia without edema, 5.2$3 &ew drugs for C3K 9pleremone--C 2ldosterone receptor blocker &esiritide

9' #hich o$ the $ollo%in! a!ents does not reduce mortality in patients %ith con!estive heart $ailure'. $igitalis----------------ans(E$! ;T+8') ". Kurosemide *. 9nalapril A. %eta blocker $iscussion%asic principle of drug therapy in C3K(3)'J ;-'AAG)'. $rugs to reduce preload T &D*, diuretic ". $rug to increase pumping of myocardium T digo in *. $rug to reduce after load T 2C9-. (9 tra Iuestion- Controversial #uestion) "' #hich o$ the $ollo%in! a!ents is contraindicated in patients %ith HF'. $igitalis ". Kurosemide *. 9nalapril A. %eta blocker-----------------ans(5ee e planation to I-*)

ardiomyopathy--C ;-'A(' 3)'J


H' ardiomyopathy is seen in all except'. ;ompeLs disease ". KriedrichLs ata ia *. 7owe syndrome--------------------------ans (/II>S 3ov (--9) A. $uchenne ?uscular dystrophy $iscussionDilated cardiomyopathy2 ;re-disposing conditions, &' /lcoholJ ". $rugs do orubicin(causes irreversible dose dependent cardiomyopathy), Cyclophosphamide1, Cocaine(also vasoconstriction--C ?.), .matinib,trastuzumab *. 2utoimmune A. ;eri- or postpartum1 +. 5elenium deficiency1 H. $uchenne myopathy1 J. KreidrichLs ata ia (. /lycogen storage disease(;ompeLs disease) B' S/> is seen in'. 3DC?---------------------ans ". Constrictive pericarditis *. $C? A. 4estrictive C? $iscussion- 52?, 5ystolic 2nt. ?ovement of ?itral valve Hypertrophic cardiomyopathy(H) >)2 '. 2utosomal dominant inheritance1 ". J8: have mutations in genes encoding beta-myosin, alpha-tropomyosin, and troponin-! *. Kamily history of sudden death may be there Symptoms = si!ns2 '. 5udden death(2lso in 25,?.,?>;) ". Verky carotid pulsation1(K2I) *. @a@ wave in V>; A. $ouble ape beat1(K2I) +. 5A heart sound H. 3arsh e-ection systolic murmur J. !he intensity of murmur increases on standing and >alsalva1(most commonly asked #uestion in world)--C also see I-'J 9cho, 52?--C 5ystolic anterior movement of mitral value1. 4 , 5eptal myomotomy--C surgical or chemical with 1alcohol(absolute alcohol) $rugs contraindicated, $igo in(absolutely), &itrates, %eta agonist, $iuretics

Pericardial diseases
&&' Pulsus paradoxus is seen in'. Cardiac tamponade--------------ans ". ;ulmonary oedema *. 5-* A. C; $iscussionardiac tamponadeSi!ns, pulsus paradoxusJ , Ewart sign is positive4, 53 is absent4, 6 descent is never prominent4

Dia!nosis, ?ec1Es triad'. Kalling %; ". 4ising V>; *. 5mall, #uiet heart E 82 Electrical alternansJ6 lo% volta!e E 8J Echo is dia!nostic2 echo-free +one around the heart &-' #hat is not seen in P( onstrictive Pericarditis)'. &ephrotic syndrome ". ;rotein losing enteropathy *. 5-* ---------------------------------ans A. Eussmaul sign $iscussionConstrictive pericarditis does not occur in rheumatic pericarditis(K2I) Clinical Keatures, !hese are mainly of right heart failure with raised V>;; Kussmauls sign is positive (V>; rising parado ically with inspiration). Prominent ! descent4, 5-3 absent, diastolic pericardial 1noc1J, hepatosplenomegaly, ascites and oedema. !he apical pulse is reduced and may retract in systole(?roadbentEs si!n) &%, $iastolic pericardial knock is also heard in early diastole. 5-* is also heard early diastole but 3arrison 'J)e !able "*"-", ;age &o. 'AG' mentions that 5-* is absent in C;. Complications'. &ephrotic syndrome1 ". ;rotein loosing enteropathy "' / ,5 yrs old male .) substernal chest pain a!!ravated by inspiration and relieved by sittin! up' He has a H.) :?' Dun! $ields are clear to auscultation6 and heart sound are some%hat distant' <A sho%s an enlar!ed cardiac silhouette' :he next step in evaluation is'. 4ight lateral decubitus film ". Cardiac catheterisation *. 9chocardiogram---------------ans A. 5erial 9C/s 9 planation- /!-(J I"(+; 3)'J ;-'A(G !he patient@s pleuritic chest pain that is relieved by sitting up is most likely d)t pericarditis. 2 percardial friction rub may initially be present, then disappear, with the heart sounds becoming fainter as an effusion develops. 7ungsounds are typically clear. 2n enlarged cardiac silhoutte without clear chest -ray findings of heart failure suggests pericardial effusion. 9chocardiography is the most sensitive, specific way of determinig whether pericardial fluid is present. !he effusion appears as an echo-free space between the moving epicardium and stationary pericardium. .t is unnecessary to perfrom cardiac catheterisation for the purpose of evaluating pericardial effusion. 4adionucleide scanning is not a preffered method for demonstarating pericardial fluid.

AHD and Endocarditis


&0' arey comb murmur is seen in'. 5evere pulmonary 3! ". 2c. 4heumatic carditis--------------ans *. ?itral stenosis A. ;ure aortic regurgitation &9' #hich is not seen in mitral value prolapse'. ;ansystolic murmur------------------------ans ". ?id or late systolic click *. Cerebral embolism A. >entricular premature contractions $.5CF55.D&- 5ystolic but not pansystolic. ;nasystolic murmur is seen in !4.

>itral valve prolapse.t is the commonest valvular lesion in the world. .t is seen in 1+-J: of the young girls. Occurs alone or with7 %5$, P$%, Cardiomyopathy, 'urner8s syndrome, "arfans syndrome # Osteogenesis imperfecta, Pseudoxanthoma elasticum, 9P9 syndrome 5igns, a. &on e-ection !id-5ystolic click4 b. 2 late systolic murmur4. ("ntensity of the murmur increases on standing and :alsalva4 ) &H' De$t ventricular hypertrophy is not seen in'. ?5-------------------------ans ". 25 *. 2ortic incompetence A. 9ssential hypertension $.5CF55.D&- (K2I) >S(3)'J ;-'AH+) Presentation, $yspnoea1 on e ertion (main symptom); hemoptysis1, 3oarseness of voice (2utner syndrome) )n auscultation, ;oud 5-14< Opening snap4&pliable valve4)< rumbling mid-diastolic murmur4, 7oud ;". /raham steel murur of ;4 and Car$ello sign &murmur is loud during inspiration) also found# Severity is indicated by- 7onger the diastolic murmur, the closer the opening snap to 2-" Echocardio!raphy dia!nostic. 4educed 9K slope is characteristic1 7>3 and 5* not found in ?5. &B' >itral re!ur!itation may occur in'. ?itral valve prolapse ". 4upture of papillary muscle *. 4upture of chordae tendineae A. 2ll----------------------------------------ans $.5CF55.D&>itral re!ur!itation(3)'J ;-'AHG)auses, a. Kunctional(7> dilatation) b. 2nnular calcification(elderly) c. 4heumatic fever d. .nfective endocarditis e# !itral valve prolapse f# =upture chordae tendinea g# Papillary muscle dysfunction>rupture h. Connective tissue disorder)(Ehlers-$anlos,!arian8s ) i. Congenital(may be associated with other defects.eg 25$,2> canal) Symptoms, 9asy fatigability1 (commonest symptom ) (-' Dancin! carotids is also 1no%n as'. 3illLs sign ". IuinckeLs sign *. CorriganLs sign----------------------ans A. !raubeLs sign $.5CF55.D&-(2..?5 ?ay "88+) /A-;eripheral signs1'. /ustin Flint murmurJ may be heard in severe 24 ". ollapsin! (water hammer) pulseJ *. orri!anEs si!n1 (carotid pulsation) A. de >ussetEs si!n1 (head nodding) +. DuroLieLEsi!n1 ($emoral diastolic murmur as blood flows backwards in diastole)

H. J. (. B'

Hill Si!n -%; difference in upper and lower limb--C (F7677) "uinc1eEs si!nJ (capillary pulsations in nail beds) :raubeEs si!nJ piston sound over femoral arteries #ide pulse pressureJ

(&' Hepatome!aly %ith liver pulsations indicates'. !4------------------------------ans ". ?4 *. ;ulmonary hypertension A. ?5 $.5CF55.D&- (2.;/ "88G) :ricuspid re!ur!itation2 auses'. Kunctional (Commonest cause of !4) seen in cor-pulmonale1 ". ;ulmonary hypertension Si!ns2 iant ?v? waves4, ;rominent ?y? descent4 in V>;, Pansystolic murmur4, arvalloEs si!n, ;ulsatile hepatomegaly1, -aundice, ascites ((' H/ E7 does not include'. 3aemophilus influenze------------ans ". 2ctinobacillus *. Cardiobacterium A. 9ikenella $.5CF55.D&- (2..?5 ?ay "88J) 32C9E ,aemophilus species &,aemophilus parainfluen+ae, ,aemophilus aphrophilus, ,aemophilus paraphrophilus) %ctinobacillus actinomycetemcomitans Cardiobacterium hominis Eikenella corrodens @ingella Commonest cause of native endocarditis--C 5.aureus(3)'J)

E 8
(,' Prolon! ": interval is seen in(>:A)'. C4K----------------ans ". Cirrhosis *. CD;$ A. &one $.5CF55.D&- (5ee I-*") 3ormal ":2 -',0--'++ secJ Prolon!ed ": intervalJ, 9lectrolyte imbalance (3ypokalemia1, 3ypocalcemia1, 3ypomagnesemia1), Class '2 anti arrhythmic drugs(Auinidine1) bradycardia, head in-ury, hypothermia, sotalol4, antihistamines, macrolides4(eg erythromycin), amiodarone 1, Phenothia+ine4, 'ricyclic1, !orse $e ;ointes(caused d)t hypokalemia)
Short ": interval, 3yperkalemia, 3ypermagnesemia, Class '% anti arrhythmic drugs, $igo in, 2cute ?.

(+' / 9, year old patient develops acute renal $ailure secondary to bladder obstruction' His serum creatinine is H m!; %ith potassium o$ 0'5 meM.D' :he patient had an electrocardio!ram6%hich revealed pea1ed : %aves as the only abnormality' It the hyper1alemia %ere not corrdcted 6%hat %ould be the expected next electrocardio!raphic abnormality'. !-wave inversion
". *. A. ;4 prolongation and ;-wave flattening------------ans ;rolongation of the I45 interval Bidening of the I45 interval

$.5CF55.D&-(2.;/ "88() K2I Hyper1aliemia , !all, tented ! wave1, ;rolong ;4, ;-Bave disappear, Bide I45 (U sine %aveL appearance) Hypo1aliemia , ! wave become smaller and then disappear, prominent F waves1, prolong ;4, 5! segment sagging.

(5' E 8 chan!es in >obitL type I is'. ;rogressive ;4 interval prolongation with a ; wave followed by dropped I45-------ans ". I45 wide with 5! and ! directed opposite to I45 *. ;4 interval is C8.H" sec A. 4 and ! phenomenon $.5CF55.D&-(Confusing #uestion) ;rogressive ;4 interval prolongation with a ; wave followed by dropped I45 (0' First E 8 chan!e in acute >I is'. 5! segment elevation ". 7oss of 4 wave *. .nverted ! wave A. !all peaked ! wave-------------------ans $.5CF55.D&- (Confusing #uestion) ;/. $ec "88H ?., Bithin minutes, the ! wave may become peaked4 (9arliest features)1 Bith "-* hrs, 5! segments may begin to rise(Pardees sign)1 Bithin (-'" hrs, the ' wave inversion4 Bithin "A-A(hrs, pathological B waves4 begin to form. I waves usually persist in old !"4 :all : is the earliest manifestation of acute ?..(K2I) /D (9' / +- year old man6 smo1er6 complains o$ epi!astric pain since an hour' )n electrocardio!raphic examination he is $ound to have S: elevations su!!estin! an in$erior %all in$raction' 3ext step in the mana!ement %ould be'. 2spirin---------------------------ans ". !hrombolytic therapy *. ;antoprazole A. %eta-blockers $.5CF55.D&- (Confusing #uestion) 2..?5 &ov "88J %est method to diagnose angina--C 3istory 2dd beta-blocker in all cases of angina if no C). e ists 5ee ne t #uestion

(H' Initial treatment o$ choice $or a patient o$ anterior %all >I %ith cardio!enic shoc1 is'. ;!C2--------------------------------------------ans ". .ntra aortic balloonpumping(.2%;) *. 5treptokinase A. $opamine drip $.5CF55.D&- 2.;/ "88( (Confusing #uestion)--C3)'J ;-'+'A ?asic concept o$ mana!ement o$ >I ( oncept o$ Aeper$usion therapy and concept o$ myocardium salva!e) >ana!ement o$ acute coronary syndrome (/ S)&' /spirin ". :hrombolysis (if no contraindication) or primary angioplasty *. %eta-blocker A. 2C9-inhibitor, =ole of %CE" in acute !", !o control %;, !o control 7>K.!o help in the remodelling of infarct tissue AolaLapine-.t is new angina drug useful for refectory angina (&ew $rug)

Extra " (/II>S >ay (--H)' Dru! used to per$orm stress E H)a. !hallium b. $obutamine-------------ans c. $opamine d. 2denosine (B' >ost important pro!nostic $actor in an in$arct case is'. 9-ection fraction------------------ans ". C;E *. >entricular tachycardia A. 7eft main coronary artery obstruction $.5CF55.D&- 7illip's lassi$ication for assessing the prognosis of a case of ?.. .t is based on degree of 7> dysfuction)Feature Class .- &o signs of pulmonary or venous congestion Class ..- Crepts at lung bases, 5*, tachypnea Class ...- 5evere heart failure, ;ulmonary edema, systolic %; C G8 >ortality +-'8: 8-"+: +8-H8:

Class .>- Cardiogenic shock, systolic %; 6 G8 mental confusion, cyanosis (8-G8: ,-' #hich dru! is used to prevent restenosis in stent'. 5irolimus-----------------ans ". Cyclosporin *. ;rednisolone A. 9rythromycin $.5CF55.D&- (&ew Iuestion) Percutaneous transluminal coronary an!ioplasty (P: /) involves balloon dilatation of the stenotic vassel(s). Stentin! reduces restenosis rates' $rug-coated stents reduce restenosis. $rug use is rapamycin4, sirolimus, paclita el(?CI)

;redisposing factors for ventricular free-wall rupture after ?. include advanced age as well as the first ?., probably d)t lack of coronary collaterals. ?ost commonly seen b)n '-A days after ?.. 7ateral and anterior walls are most often involvedand it is typically seen with large ?. involving C"8: of the ventricle. 4upture typically occurs at the -unction of infarct with the normal tissue. ;atients are diagnsed by 9C/ and treatment involves operative treatment if possible.

/rrhythmias
,&' / +H year man is admitted to N %ith an acute in$erior >I' :%o hours a$ter admission6 his ?P is H0.5( m!H!O his pulse is +-.mt %ith sinus rhythm'#hich o$ the $ollo%in! %ould be the most appropriate initial therapy'. .mmediate insertion of a temporary transvenous pacemaker ". .)> administration of atropine sulfate, 8.H mg--------------------ans *. 2dministration of normal saline, *88ml over '+ min A. .)> administration of dobutamine, 8.*+ mg)min $iscussion- (2..?5 &ov "88() 2tropine--C unsucessful--C 5aline ,(' :orse-de-pointes is caused by'. 3ypermagnesemia ". 3ypokalemia-------------------ans *. 3yperkalemia A. 3ypercalcaemia $.5CF55.D&-

3ypokalemia (commonest cause) ;rolonged I!c interval1 9C/ oscillates along baseline--C

Causes- Common causes for torsades de pointes include diarrhea, hypomagnessemia and hypokalemia. $rug interactions such as erythromycin or mo iflo acin, taken concomitantly with inhibitors like nitroimidazole, dietary supplements, and various medications like methadone, lithium, tricyclic antidepressants or phenothiazines may also contribute. !reatment- >a!nesium sul$ate

ardiac 3eoplasm
,,' :rue statements re!ardin! cardiac neoplasms include'. 7ymphoma is the most common malignant neoplasm that primarily involves the heart ". !he most common site for a myoma is the right atrium *. ?y omas may arise as part of a familial syndrome that also includes pigmented skin lesions and endocrine abnormalities--------------------ans( A. 2 midsystolic YplopZ typically indicates the presence of a cardiac my oma $.5CF55.D&,(3)'J ;-'AG+)- Cardiac my oma is a rare benign cardiac tumour in 1left atrium usually sporadicJ, may be $amilialJ (autosomal-dominant)J' .t may mimic infection endocarditis (fever, weight loss, clubbing, raised 954), or mitral stenosis4 (left atrial obstruction, systemic emboli, 2f). 2 Utumour plopL may be heard, and signs may changed according to posture. 3istologically they are composed of stellate cells in a loose my oid background. .n contrast Ahabdomyomas are the most common primary cardiac tumors in infants and children and often occur in a)w tuberous sclerosis. 3istologically the so-called spider cells may be seen. ;apillary fibroelastosis usually are incidental lesions found at the time of autopsy and are probably hamartomas rather than true neoplasms. arney complex comprises'. my omas (cardiac1, skin, and)or breast), ". lentigines and)or 1pigmented nevi, and *. endocrine overactivity (primary nodular adrenal cortical disease with or without Cushing@s syndrome, testicular tumors, and)or pituitary adenomas with gigantism or acromegaly). Certain constellations of findings have been referred to as the &2?9 syndrome (nevi, atrial my oma, my oid neurofibroma, and ephelides) or the 72?% syndrome (lentigines, atrial my oma, and blue nevi), although these likely represent subsets of the Carney comple . !ests, 9nchocardiography !reatment, 9 cision

Aheumatolo!y
,+' :3F- alpha is involved in the patho!enesis o$ %hich o$ the $ollo%in! disorder'. 42-------------------ans ". 579 *. ;soriasis A. &one $.5CF55.D&-(?aheshwari ;-"AA) !&K-alpha is involved in the pathogenesis of 42 Dia!notic criteria o$ A/- Four of seven criteria are re#uired1 '. !orning stiffness- lasting ' hour before ma imal improvement ". %rthritis of 3 or more Coint areas -'A possible -oint areas are right or left ;.;, ?C;, Brist, 9lbow, Enee, 2nkle, and ?!; -oints

*. A. +. H. J.

2rthritis of hand -oints 5ymmetrical arthritis 4heumatoid nodules1 ;ositive 5erum rheumatoid factor1 4adiographic changes- erosions(3allmark) or une#uivocal bong decalcification localized in or most marked ad-acent to the involved -oint.

%ackache never a symptom of 42....... ,5' False positive Aheumatoid $actor can be associated %ith all except'. .nflammatory bowel disease--------------ans(5ee I-*J) ". 3bs2g *. >$47 A. Coombs test--C diagnose 579 $.5CF55.D&7aboratory findings in 42- 42 factor(.g?) is positive in J+: patients of 42 though it is found in +: of healthy persons. 2nti P antibody are also seen. !he presence of 4heumatoid factor in 42 correlates with e traarticular menifestation of the disease

)ther conditions in %hich A/ $actor is positive are- SDE, 5-ogrenLs syndrome, .nterstitial pulm fibrosis, Hepatitis ?, 7eprosy, 5ubac. bacterial endocarditis, Chronic liver disease, 5arcoidosis, .?&, !uberculosis, Syphilis,>isceral leishmaniasis, ?alaria :reatment o$ A/(7D: P-(-()'. &52.$5- !o control the symptoms and signs of local inflammatory process. !hese drugs have minimal effect on the progression of the disease. ". $isease modifying antirheumatic drugs- !hese drugs alter the course of 42. !hese drugs have minimal effect on the inflammation. 5o are not useful as analgesics. !hese drugs should be used early1 in the course of the disease as they slow the progression of disease. !he most commonly used $?24$ is ?ethotre ate. '. Immunosuppressants- !he immunosuppressive drugs used are ?ethotre ate, 2zathioprine, leflunomide, cyclosporine and cyclophosphamide ". Dther drugs- d-penicillamine, and sulfasalazine, /old , 3ydro ychloro#uine *. %iological response midifiers&' :3F alpha neutraliLin! a!ents'. >onoclonal antibody to !&K alpha (In$liximab) ". !&K-alpha type [.. receptor antagonist (Etanercept)(&ew $rug) *. /dalimumab (3uman antibody to !&K)(&ew $rug) ". ID-& 4eceptor antagonist T 2&2E.&42 I (&ew $rug) A. 2d-uvent drugs- Corticosteroids +. /ene therapy- /enes which intercept the pathway of inflammatory cascade reaction are used H. 5urgery- 5ynovectomy, tenosynovectomy, arthroplasty and total -oint replacements ,0' Aituximab (/nti- D (- antibody) is used in all except'. 579 ". 42 *. ;&3----------------ans A. &37 $.5CF55.D&- (2..?5 &ov "88() Aituximab(anti D(- antibody) --C C77 also ,9' Serone!ative arthritis include'. 25 ". 4eiterLs arthritis *. ;soriatic arthritis A. all------------------------ans $.5CF55D&-

Spondylo-/rthrosis, .t is a group of disease which include'. 2nkylosing spondylitis ". 4eiterLs disease *. 4eactive 2rthritis(Mersinia,5almonella,/onococcus) A. ;soriatics arthritis +. Vuvenile rheumatoid arthritis(V42)Wnot adult rheumatoid arthritis which is $BA)$4AX Keatures, '. 5eronegativity ". 372 ?(9 associated *. 2 ial arthritis, ;athology in spine (5pondylo-)and sacroilias (5.) -oints(%ackache is the presenting symptom) A. 2symmetrical large--oint oligoarthritis(i.e. 6+ -oints) or monoarthritis +. 9nthesitis, .nflammation of the site of insertion of tendon or ligament into bone H. 9 tra-articular manifestations eg. anterior uveitis, aortic regurgitation, CrohnLs or FC AEI:EAES DISE/SE(Imp)2 .t is characterised by a triad of1# 5eronegative oligoarthritis4 2# ConCunctivitis *. Donspecific urethritis, '-* weeks following bacterial dysentery(epidemic form) or e posure to se ually transmitted disease 2rthritis occurring alone following se ual e posure or enteric infection is known as reactive arthritis. 4eiterLs disease can occur in epidemic1 form(K2I) 2..?5 &ov "88( 9 am Etiolo!y- 9nterpathogenic %acteria in 4eactive 2rthritis 5almonella,5higella,Campylobacter,Mersinia Chlamydia linical Features'. .t presents with monoarthritis of a knee or an asymmetrical inflammatory arthritis of interphalangeal -oints ". ;atient can have heel pain.2chilles tendinitis or plantar fasciitis with presence of circinate balanitis. (;resence of rash of keratoderma blennorrhagia (2..?5 &ov "88() is diagnostic of 4eiterLs disease in the absence of classical traid.)--C Ck printed matter *. Dcular involvement (mild bilateral con-unctivitis) 9 tra #uestion"' ircinate balanitis is seen in(/IP8 (--9)'. 42 ". 579 *. 4eiterLs disease----------ans A. /out "' #hat is not seen in AeiterEs syndrome(/II>S 3ov (--H) '. 5ubcutaneous nodules---------------------ans ". Eeratoderma blennorrhagicum *. Circinate balanitis A. Dral ulcers ,H' Schirmer test is positive in'. 5-ogrenLs syndrome------------------ans ". %ehcetLs syndrome *. 579 A. BhippleLs disease DiscussionSF)8AE3ES SI3DA)>EDe$inition- Chronic, slowly progressive autoimmune disease characteried by lymphocytic infiltration of

the e ocrine glands resulting in erostomia and dry eyes. Causes of secondary 5-ogren@s syndrome'. 4heumatoid 2rthritis(most common cause) ". 579 *. 5cleroderma A. ?i ed connective tissue disease +. ;rimary biliary cirrhosis H. >asculitis J. Chronic active hepatitis (. ;olymyositis G. 3ashimoto@s thyroiditis '8. .nterstitial pulmonary fibrosis 2ntibodies to Ao.SS-/ and Da.SS-? anti!ens; 2ntibodies to alpha-$odrin--C a salivery gland specific protein &37 common in 5-ogren ans also in 3.>........ linical >eni$estations(:riad)'. $ry eye ". $ry mouth *. %)7 ;arotid enlargement ,B' Sudden renal $ailure occurs in'. 579 ". ;55---------------------ans *. 42 A. 25 $.5CF55.D&SIS:E>I S DEA)SIS $efinition- !his is a generalised disorder of connective tissue characterised by fibrosis and degenerative changes in the skin (scleroderma) and many internal organs $iagnosis 2ntibodies seen are'. %ntinuclear antibodies(2&2) ". 2ntibodies to single stranded 4&2 3# %nti-5cl-E/ ;art of Crest 5yndrome(2nticentromere 2b) +-' #hich radiolo!ical $eature is most important to help di$$erentiate rheumatoid arthritis %ith SDE'. 9rosion---------------------------ans ". Vu ta articular osteoporosis *. 5ublu ation of ?C; -oint A. 5welling of ;.; -oint $.5CF55.D&Daboratory $indin!s in a cause o$ SDE 2ntibodies. !he antibodies seen in 579 are'. 2ntinuclear antibodies(2&2)- ?ost sensitive test ". 2nti ds D&2- most speci$icJ for 579. 3igh titres are associated with nephritis and disease activity. *. 2nti 5m [ 2lso specific for 579 A. 2nti- 4o (55-2) [ 2ssociated with subacute cutaneous lupus and %D% negative lupus +. 2nti-7a (55-%) [ 4isk for nephritis is low if present H. 2nti-histone- ?ore fre#uent in drug induced 79. J. N&A3P ab- 5een in mi connective tissue disorder. Bhich include 579,;55 and polymyositis +&' >ost speci$ic dia!nostic $indin! in !out is'. 4aised serum uric acid

". Fric acid crystals in urine *. ;resence of monosodium urate crystals in synovial fluid under polarized microscopy---------ans A. ;resence of calcium pyrophosphate crystals in synovial fluid under polarized microscopy $.5CF55.D&8out'. >ales C females. 8reat toe, /rade + tenderness ". ;resence of monosodium urate crystals in synovial fluid under polarized microscopy (5erum uric acid level may be normal) *. Kor acute pain [ colchicine, indomethacin (%ut aspirin is contraindicated) A. Kor prophyla is- 2llopurinol +. Fricosuric drugs- ;robenecid, 5ulphinpyrazone +(' -/3 / /b are seen in'. Begener@s granulomatosis----------------ans ". ;2& *. Chrug-5trauss A. 579 $.5CF55.D&-(2..?5 ?ay "88J) -/3 / antibodies are seen in #e!enerEs !ranulomatosis these are directed against proteinase-, ;-2&C2 antibodies are seen in ?icroscopic polyangitis, Churg-5trauss syndrome, Crescentic glomerulonephritis, /oodpastureLs 5yndrome +,' A/S is seen in all except'. ;2&------------------------------ans ". !akayasu *. 2therosclerosis A. Kibromuscular dysplasia ++' :rue about !iant cell arteritis are all except'. 3igh dose steroid is drug of choice ". 954 is usually elevated *. .nternal carotid artery is particularly susceptible A. ?ainly affect people below age of A8 years-----------------------ans $.5CF55.D&:emporal arthritis(;olymyalgia rheumatica fre#uently occur with !emporal arteritis) ?ainly effect people above age of H8 years 3eadache .nternal carotid artery is particularly susceptible >isual disturbance can occur (affects retinal 2.--C branch of .nt. Carotid) 954 is usually elevated 3igh dose steroid is drug of choice +5' /ll o$ the $ollo%in! are true about 7a%asa1i disease except'. 4ash ". 7ymphadenopathy *. 9 tremities edema A. ;urulent con-unctival congestion----------------ans $.5CF55.D&5- (2.;/) 3)'J ;-"'*8 '. 4ash ". 7ymphadenopathy (does not respont to antobiotics) *. 9 tremities edema A. Con-unctival congestion it is non-purulent 4 , .>.g +0' /nti Fo-& antibody = /nti synthetase antibody are seen in'. 5ystemic lupus erythematosus

". 5ystemic sclerosis *. $ermatopolymyositis----------------------ans A. $ermatitis herpetiformis $.5CF55.D&.n polymyositis eye muscle are not involved and biopsy is confirmatory. +9' >ali!nant A/ occur i$ there is severe involvement o$'. 9ye ". 5kin------------------ans *. &erve A. Voint $iscussion- .t is a very simple and easy #uestion\\\\

3ENA)D)8I
4efresh your knowledge of neuroanatomy to understand &eurology much better....

erebral Hemisphere
+H' / ,- year6 male c.o headache $or last & year = !radually pro!ressive paraparesis $or last & month' >ost li1ely cause'. 2C2 infarct ". ?eningioma----------------ans *. !% spine A. ?C2 infarct (Extra "uestion)' > / in$arct leads to contralateral'. 7eg paralysis ". 2rm paralysis *. %oth------------------ans A. &one +B' #hich statement re!ardin! 8erstmannEs syndromeJ is true'. 2ffected patients have difficulty distinguishing right from left----------------ans ". .t results from a lesion of the nondominant parietal lobe *. ?otor deficit is a prominent feature A. 2pra ia is common $iscussion- ($;/ "88H) Parietal lobe/' Dominant parietal lobe Disease, /artsman syndrome1 Features2 '. 2calculia ". 2graphia *. Kinger anomia(can not name finger) A. $ifficulty in ri!ht and le$t differentiation Extra points,

Frontal lobe Disease, Koster Eennedy syndrome1. .psilateral optic atropy and contralateral papilledema. .t results from the simultaneous presence of raised intracranial pressure and optic nerve compression secondary to tumor [ classically due to a meningioma of the olfactory groove or more commonly due to a meningioma of the sphenoid wing. )ccipital lobe disease, Cortical blindness1

:emporalJ lobe disease, Eluver %ucy syndrome1(%ilateral temporal lobe involvement)

5-' I$ #ernic1eEs area is dama!ed in the dominant hemisphere6 it %ill result in'. .rrelevant and rapid speech------------------------------------ans ". 5enseless speech and difficulty in outflow of words *. 5peech with difficulty in outflow of words A. 5peech with difficulty in naming ob-ects $iscussionSpeech/natomy = Physiolo!y-

%rocaLs area T 7ies in dominant inferior frontal gyrus1. .t is motor speech area(5peech outflow is disturbed in in-ury) BernickeLs area T 7ies in dominant temporal lobe1. .t is sensory speech area 2rcuate fibres T !hey connect the two speech areas

Physiolo!y o$ speech2 5poken words --C /o to ears --C /o to auditory area in temporal lobe via (th nerve--C /o to BernikeLs area where speech is understood--C ?essage go to %rocaLs area via 2rcuate fiber--C %rocaLs area is a motor speech which gives command to vocal cord and lip area in the motor corte --C speech is spoken Diseases2 %rocaLs area involvement--C 5peech out flow and fluency is lost BernickeLs area involvement--C 5peech understanding is lost. ;atient speaks senselessly 3ominal aphasia(/nomic aphasia) is seen in metabolic encephalopathy' It is an early $eature o$ /lLeimerEs diseaseJ Nrinary bladder-

/natomy 3erve supply 5ympathetic- !''-7" (3ypogastric &)--C 5upply !rigone ;arasympathetic- 5", 5*, 5A (;elvic &)1--C 5upply $etrusor 5omatic [ 5", 5*, 5A (;udendal nerve)1--C 5upply Ext' Sphincter Physiolo!y, 2fferent by all three nerves i.e. ;elvic, 3ypogastric, ;udendal nerve. 9fferent mainly by pelvic nerve. $etrussor muscle is uni#ue--C does not follow starling@s law. 2rea of social behaviour and personality lies in $rontal lobe.
:ypes o$ urinary bladder disturbance2 '. Automatic%&ypertonic bladder%Complete spastic , 7esion in the spiral segment above 5",5*,5A. 5mall capacity bladder, Kre#uent voiding at interval (%ladder start behaving like a new born bladder.) ". Autonomous%&ypotonic bladder- 17esion at 5", 5*, 5A, 7arge capacity bladder, $istention with overflow *. Cortical%'ninhibited Bladder- Frontal -.nappropriate micturition, loss of social inhibition control

5(' Painless burn in hand is a characteristic $eature o$'. 5yringomeylia-------------------ans ". !halamic syndrome *. Cord compression A. 579 $iscussion- (3)'J ;-"+GA; "+(G Kigure *J"-')

5,' #hich is not a $eature o$ extramedullary tumor'. 9arly corticospinal signs and descending paralysis ". 4oot pain or ?idline %ack-;ain *. C5K 2bnormal A. 5acral sparing--------------------ans $iscussion- (2.;/ "88() omparison o$ intramedullary and extramedullary comparisonSymptoms '. 4oot pains ". >ertebral pain *. ?otor Beakness A. 5arcral .nvolvement Si!ns '. %rown 5e#uard ". $issociative sensory loss Investi!ation Fncommon ;resent Common 2bsent 7ate Intramedullary Fncommon Nncommon 7ate Date ExtramedullaryJ Common ommon 9arly Early--C Commonly involved

*. %ladder)bowel dysfunction 9arly

5ubarachnoid %lock

7ate Sless marked 9arly S marked

Froin syndrome - alteration in the cerebrospinal fluid due to block, which is yellowish and coagulates spontaneously in a few seconds after withdrawal, owing to its greatly increased protein (albumin and globulin) content; noted in loculated portions of the subarachnoid space isolated from spinal fluid circulation by an inflammatory or neoplastic obstruction. 5yn, loculation syndrome.

5+' #hich $indin!s are $ound in ?ro%n SeMuard syndromeIpsilateral loss ' " * A >ibration and motor >ibration and ;ain ;ain S vibration ontralateral loss ;ain and temperature ?otor S temperature !emp. and motor

;ain and temperature >ibration and motor

2nswer- "

?eevor's si!n- 7esion at !G-!'81 paralyse the lower but not the upper abdominal muscles, resulting in upward movement of umblicus when abdominal wall contracts (beevor@s sign)

"' /n H- yrs old male %ith mid alLheimer's disease has been started on donepeLil 5 m! a$ter he continued to have di$$iculty in $inancial matters and 1eepin! trac1 o$ the dayo$ the %ee1 and time' /$ter , months the $amily $eels that there has been no improvement' :here are no complaints o$ nausea6 diLLiness or hypotension' :he patients %i$e $eels the medicaton is unnecessary' :he best advice is'. $iscontinue the donepezil ". .ncrease the donepezil dose to '8 mg *. Continue donepezil to prevent further pla#ue formation A. Continue donepezil for * to H months and reevaluate mental status 9 planation- /!-(J I-'JA !he best course would be to continue the donepezil and see if it slows progression of cognitive function loss based on ??59 or family assessment. !he success of the intervention needs to be evaluated over a longer time period realising that success may mean maintainting baselinefunction. !he anticholinesterase inhibitors do not prevent pla#ue formation. .ncreasing dose is rarely helpful and often causes side effects. !here is no datato suggest that one cholinesterase inhibitor works better than another.

ranial nerves = ?rain stem


50' /ll are true about ?enedictEs syndrome except'. Contralateral choreoathetosis ". $ense hemiplegia------------------ans *. 7esion is in mid brain A. .psilateral *rd nerve palsy $iscussion-(2.;/ "88J) 3)'J ;-"+"+; ?E-. ;-*8" 5ite--C >idbrain2 ;C2 Dcclusion#eber's synd &othnagel syndrome %enedikt@s syndrome C)7 hemiparesis O ipsilateral *rd nerve palsy C)7 cerebellar signs O ipsilateral *rd nerve palsy C)7 cerebellar signs O ipsilateral *rd nerve palsy

Claude syndrome G *enedikt?s H Dothnagel syndrome 3?- &othnagel syndrome--C 5ame presentation as %enedict@s

%ilateral infarction in distal ;C2 produces cortical blindness (blindness with preserved pupillary light reaction. !he patient is often unaware of the blindness or may even deny it (Anton(s )yndrome ) 59' >illard 8ubler Syndrome consists o$ the $ollo%in! E< EP:'. .psilateral +th Cranial &erve ;alsy--------------ans ". .psilateral Hth Cranial nerve ;alsy *. .psilateral Jth Cranial &erve ;alsy A. Contralateral hemiplegia $iscussion5ite, Pons Koville@s syndrome(dorsal pontine in-ury) T lat' !aLe palsy6 ipsilateral >., >.., palsy, C)7 hemiparesis >illard-8ubler synd (ventral pontine in-ury) T ipsilateral >., >.. palsy C)7 hemiparesis 5H' / ,- year old male patient presented %ith h.o diLLiness6 verti!o6 dyspha!ia6 alon! %ith Horner syndrome on the same side' Doss o$ pain and temperature sensations on the le$t side %as noted' :he artery most li1ely to be involved in the condition described above is'. 2nterior inferior cerebella ". ;osterior inferior cerebella---------------ans *. ?iddle cerebral A. 5uperior cerebellar $iscussion- (2..?5 ?ay "88H)--C Ballenberg@s 5yndrome--C ;.C2 involved--C Classic findings 5B' / +5 year old man presents %ith a daily headache' He has t%o attac1s per day over the past , %ee1s6 associated %ith tearin! and reddenin! o$ his ri!ht eye as %ell as nasal sti$$ness' :he neurolo!ic examination is non $ocal' :he most li1ely dia!nosis is'. ?aigraine headache ". Cluster headache----------------ans *. !ension headache A. %rain headache $iscussion;recipitated by alcohol ! , D ygen ;rophylla is, 7i, >alproate, 2mitriptaline (;ropranolol not effective)

@/ (3)'J- "+'*)
0-' >ost common cause o$ subarachnoid haemorrha!e is'. 3ypertension ". 2.>. ?alformation *. %erry aneurysm A. &one--------------------ans $iscussion- 3)'J ;-'J"H, :rauma is the m)c cause. 2mong options--C * 0&' :he site o$ cerebral hemorrha!e in hypertension are all except'. ;ons ". .nternal capsule-----------------ans *. ;utamen A. !halamus $iscussion- 5ite of hypertensive bleed are- Cerebellum, ;ons, ;utamen, !halamus

N>3 = D>3 Desions


0(' N>3 lesion is characteriLed by'. Beakness S spasticity-------------------ans ". Kasciculations *. 4igidity A. 7ocalized muscle atrophy

Diseases causin! dementia


0,' /lLheimerEs disease typically a$$ects'. &ucleus basalis of ?eynert-----------------ans ". 2mygdala *. ;arietal corte A. %asal ganglion $iscussion- (2..?5 &ov 8J) Controversial Iuestion /lLeimer's Disease- (;-A8H 4D2?5) ;athology, 2t autopsy, the most severe pathology is usually found in the hippocampus, temporal corte , and nucleus basalis o$ >eynert(lateral septum) ?icroscopically- neuritic senile pla#ues and cytoplasmic neurofibrillary tangles. !he neuritic pla#ue contain %-beta amyloid, proteoglycans, 2po 9, 2lpha-' antichymotrypsin !acroscopically- $iffuse atropy of cerebral corte with secondary enlargement of ventricular system %iochemically- $ecrease in cerebral cortical levels of %cetylcholine, choline acetyltransferase and nicotinic cholinergic receptors. 4eduction in norepinephrine levels in brainstem nuclei. :reatment2 !acrine $onepezil 3e%er dru!s, Aivasti!mine 8alantamine >emantine

$rugs which reduce progression of 2$ 9strogen replacement therapy1 /inkgo &52.$1 5elegiline >itamine 9

0+' :riad o$ normalPpressure hydrocephalus includes'. !remor, aphasia dementia ". 2ta ia, aphasia, gait disorder *. /ait disorder, urinary incontinence, dementia---------------------ans A. /ait disorder, urinary incontinence, lower cranial nerve palsy $iscussion- (3)'J -"+AH) &D4?27 ;4955F49 3M$4DC9;327F5 .t is a triad of abnormal gait &%taxic gait), dementia and urinary incontinence 2 type of communicating hydrocephalous with patent a#ueduct of sylvius &;3 is caused by obstruction to normal flow of C5K over the cerebral conve ity and delayed absorption into the venous system 3)D conditions producing scarring of basilar meninges such as meningitis, 523 and head trauma 9nlarged lateral ventricles (3ydrocephalus) with little or no cortical atropy. C5K- ;ressure is in high normal range. !7C, $7C, ;rotein, 5ugar are normal !reatment- >entricular peritoneal shunt Extra Muestion(/IP8 (--9)- #hich o$ the $ollo%in! disease doesnEt have autosomal dominant mode o$ inheritance'. ?arfanLs syndrome ". ;olycystic kidney disease *. KabryLs disease------------------ans

A. 3untingtonLs disease $iscussion- 2pha-galactosidase 2 deficiency. R-linked recessive lysosomal storage disease 4 , 9nzyme replacement therapy /hent@s criteria is used for the diagnosis of ?arfan@s syndrome

Par1insonism (3)'J ;-"+AG)


05' :rue about Aoti!otine'. .t is a non ergot alkanoid-----------------ans ". .t is a ?2D [ % inhibitor *. .t is an anti-o idant A. none $iscussion3e%er dru!s Er!ot al1aloid2 %romocriptine, Cabergoline, ;ergolide 3on-er!ot al1aloid, ;ramipa ol, 4opinirol, 4otigotine ? of !remors- 2ntimuscarinics--C ;rocyclidine, %enztropine, %enzhe ol 4ead whole drug clssification from ;-A'+ E$! 3europrotective therapy'. Chronic use of &52.$5 or the use of estrogen replacement in postmenopausal women may delay or prevent the onset for ;$(also 2lzeimer@s) ". 5elegiline monotherapy delayed the need for levodopa therapy *. Coenzyme I'8, an antio idant and a cofactor of comple . of the mitochondrial o idative chain, has been shown to have neuroprotective effects

Demyelinatin! disorders
00' :he most common presentin! $indin! o$ multiple sclerosis is'. .nternuclear ophthalmoplegia ". !ransverse mylitis *. Cerebellar ata ia A. Dptic neuritis------------------ans $iscussion- 3)'J ;-"H'' 9 cercise induced weakness is a characteristic symptom of ?5 ,eat sensitivityo 2ppearance of new symptoms or the worsening of pre-e isting symptoms on e posure to heat o Conduction block which may occur in response to increase temperature or metabolic derangements *hermittes symptom- /eneration of ectopic impulse giving rise to 7hermitteLs 5ympt, paro ysmal symptoms or paresthesias .nvestigations'. >AI--* $awson8s finger sign- 7esion appear to e tent from the ventricular surface, corresponding to a pattern of perivenous demyelination ". 9voked responses- !o detect slowed or absent conduction in visual, auditory, somatosensory or motor pathways *. C5K ?ononuclear pleocytosis, !7C usually 6"8 "ncrease "g 1

Oligoclonal banding4

!reatment of 2cute 2ttack'. /lucocorticoid treatment ". ;lasma e change 3# !ethotrexate4 A. .> .g +. ;ulse cyclophosphamide,2zathioprine H. ;ulse ?ethyl ;rednisolone &ew $rugs for prophylaxis, /latiramer acetate ?ito antrone .K&-beta-'a)'b &atalizumab

Epilepsy = Syncope
09' #hich o$ the $ollo%in! %ould help exclude the dia!nosis o$ seiLure in a patient %ith sudden loss o$ consciousness'. 2 brief period of tonic [ clonic movements at the time of falling ". 2n aura of a strange odor before falling *. 5udden return to normal mental function upon awakening, though with a feeling of physical weakness-------------------------ans A. Frinary incontinence $iscussion- (2.;/ 8H) ;t. is usually confused following seizure in comple partial seizure EPIDEPSI (3-'J ;-"AG()5eizure- ;aro ysmal event due to abnormal,e cessive and hypersynchronous discharges from an aggregate of C&5 neurons.?anifestation of a seizure can include impairment or loss of consciousness and sensory,motor or behavioural abnormalities. !he term epilepsy describes a syndrome characterised by recurrent sei+ures. D/SSIFI /:I)3'. ;artial seizures, i. 5imple partial--C no alteration in consciousness ii. Comple partial--C transient inability to maintain normal contact with environment. iii. ;artial seizures with secondary generalization ". ;rimary generalised seizures, i. 2bsence (;etit mal) ii. !onic-clonic(/rand mal) iii. !onic iv. 2tonic v. ?yoclonic *. Fnclassified, i. &eonatal 5eizure ii. .nfantile 5pasm A. 5tatus epilepticus ;24!.27 59.]F495- 5eizure activity is restricted to discrete areas of cerebral corte .;artial seizures are often associated with structural abnormalities of the brain..n such case C!)?4. of brain is mandatory. '. )imple partial sei+ure- !he consciousness is fully preserved during the seizure simple partial seizure cause motor,sensory autonomic or psychic symptoms.

i. Fac1sonian march- ?ovements begin in a very restricted region and gradually progress
to include a large area(not whole area). ii. 'odd?s paralysis- 7ocalised paresis in the involvement region--C followed by complete revival iii. 5eizure partialis continua- 5eizure activity continue for hours to days. ". Complex partial sei+ure - %egins with aura. Kocal seizure activity accompanied by a transient impairement of patient8s ability to maintain normal contact with the environment *. ,artial sei+ures -ith secondary generali+ation - ;artial seizure can spread to involve both hemisphere and produce generalised seizure. .t is fre#uently observed following simple partial sei+ure# /9&9427.59$ 59.]F495- 2rise from both cerebra simultaneously .4esult from cellular ,biochemical or structural abnormalitites Phases o$ a tonic-clonic seiLure, ;rodromal phase, !onic phase, Clonic phase, ;ost-ictal phase 0B' Dru! o$ choice in absence seiLure'. >alproate---------------ans ". ;henytoin *. Carbamazepine A. !opiramate $iscussion- (2..?5 8()--C $rug table 3)'J Chapter-*H*(!able *H*-() 4emember--C 5aare dard ki ek dawa hai >alproate

>enin!itis = Encephalitis
9-' :he least common complication o$ menin!itis is'. 3ypernatremia--------------ans(3yponatremia--C d)t 5.2$3) ". 3ydrocephalus *. %rain abscess A. $eath $iscussion- 3)'J ;-"H"' Causes 5. ;neumoniae C &. ?eningitidis Children- 5. 2ureus (;-"H"") Complications of ?eningitis- 4aised .C;, Cranial nerve palsy, >asculitis, Kocal deficit, 5eizure, 3ypocephalus, 5.2$5, %rain abscess, ?ental retardation, $eath, 5ubdural effusion(very common in children with 3. .nfluenza infection) 9&' #hich is not true about pro!ressive multi $ocal leucoencephalopathy'. $egeneration of cortico cerebellar system---------------ans(Ck " in copy) ". $ifficulty in speech *. .ndolent course over few years A. %rain biopsy is definitive diagnosis $iscussion- 3)'J ;-"H*A ;rogressive ?ultifocal 7eukoencephalopathy ;rogressive multifocal leukoencephalopathy(;?7) is a rare demyelinating C&5 disorder caused by the reactivation of .C $irus (VC>--C a polyomavirus) !he virus stays latent in the kidneys and lymphoid organs until reactivation ;?7 mainly occurs in adults with impaired cell-mediated immunity, especially 2.$5 patients but also in those with lymphoproliferative and myeloproliferative disorders VC> causes lytic infection of oligodendrocytes in the -hite matter 5ymptoms include altered mental status, aphasia, ata ia, hemiparesis or hemiplegia and visual field disturbances. 5eizures occur in about '(:. C5K--C &ormal 4 , 4isperidone(;-A"+ E$!) and mirtazapine(;-A*G E$!)

9(' :rue about /moebic menin!oencephalitis include'. 2cute form of meningoencephalitis ". Common in tropical countries *. Keco-oral mode of transmission A. !rophozoites in cerebrospinal fluid is diagnostic--------------ans $iscussion- 3)'J ;-'"JG .nfection caused by the ameba 3ae!leria $o%leri can also cause acute meningoencephalitis (primary amebic meningoencephalitis), wereas that caused by %canthamoeba and *alamuthia more typically produces subacute or chronic granulomatous amebic meningoencephalitis. 3ae!leria thrive in warm, iron-rich pools of water, including those found in drains, canals, and both natural and human-made outdoor pools. .nfection has typically occurred in immunocompetent children with a history of swimming in potentially infected water. !he C5K, in contrast to the typical profile seen in viral encephalitis, often resembles that of bacterial meningitis with a neutrophilic pleocytosis and hypoglycorrhachia. ?otile trophozoites can be seen in a wet mount of warm, fresh C5K. &o effective treatment has been identified, and mortality approaches '88:.

>3D
9,' >otor neuron disease is characteriLed by'. 5ensory loss ". 4igidity *. Kasciculation---------------ans A. Kocal seizure $iscussion- 4iluzole (&ew drug)--C 3)'J ;-"+J"--C read about it

>yasthenia !ravis
9+' 3ot a characteristic $eature o$ myasthenia !ravis'. 5ometimes spontaneous regression ". ;ro imal muscle involvement *. 2bsent deep refle es----------------------ans A. Katigue with e ertion $iscussion- 3)'J ;-"HJ" .n ?/, the fundamental defect is a decrease in the number of available 2Ch4s at the postsynaptic muscle membrane. 7aboratory testing'. 9drophonium chloride(!ensilon)--C 5creening test ". 2nti-2Ch4 radioimmunoassay, ^(+: positive in generalised ?/; +8: in ocular ?/. $efinite diagnosis if positive but negative result does not e clude ?/. ^A8: of 2Ch4 antibody-negative patients with generalised ?/ have anti-?u5E antibodies *. 4epetitive nerve stimulation, decrement of C'+: at *3z, highly probable A. 5ingle-fibre electromyography , confirmatory, but not specific 4efle es &ormal--C ?/, ;arkinson@s disease 95' In myasthenia !ravis6 $ace appearance is'. 5narling----------------ans ". ?ouse like *. ?ask like A. &one $iscussion, ?ouse like face--C C4K

AESPIA/:)AI SIS:E>

)bstructive.Aestrictive Diseases
99' FE@&.F@ ; is reduced in all except'. Chronic bronchitis ". %ronchial asthma *. .nterstitial lung disease----------------ans A. 9mphysema $iscussion)bstructive FE@-& F@ $ecrease decrease Aestrictive decrease $ecrease

Aatio $ecrease 3ormal.Increase decrease--C small decerease $ecrease--C %ig $ecrease 9H' In an emphysematous patient %ith bullous lesion %hich is the best investi!ation to measure lun! volume'. %ody plethysmography--------------ans ". 3elium dilution *. !rans diaphragmatic pressure A. $7CD $iscussion- (2..?5 &ov 8() 4>, K4C, and !7C- Cannot be measured by spirometry because they include the volume of gas present in the lungs !wo techni#ues of commonly used to measure these volumes, helium dilution and plethysmo!raphy !he helium dilution method may underestimate the volume of gas in the lungs if they are slowly communicating airspaces, such as bullae. .n this situation, lung volumes can be measured more accurately with a body plethysmograph, a sealed bo in which the patient sits while panting against a closed mouth piece 9B' First to be seen in acute severe asthma is'. ;ulsus parado us--------ans ". 3ypercapnia *. 2cidosis A. Cyanosis $iscussion- (2..?5 ?ay 8() 3)'J ;-'+GH Keatures of Severe attack, '. .nability to speak complete sentence in one breathe4 2# Pulse -12/ per min# *. 4espiratory rate C"+)min .# Pulsus paradoxus of -1/ mm,g I# PEF between 33 to I/J of predicted Keatures of Di$e-threatenin! attack1# 5ilent Chest4 2# Cyanosis4 3# *radycardia4 A. Confusion4 (altered sensorium) I# PEF 0 33J of predicted4 H-' False re!ardin! chronic bronchitis is'. 3emoptysis ". C" consecutive years *. ;roduction cough

A. Cough for * months in a year---------------ans(2.;/ @8H)--C * consective months $iscussion- 3)'J ;-'H*G H&' SIAS includes all except'. 3ypothermia ". 7eukopenia *. !achycardia A. 3ypotension-----------------ans $iscussionDe$initions used to describe the condition o$ septic patients %acteremia 5epticemia Systemic in$lammatory response syndrome (SIAS) ()nly this is Imp') ;resence of bacteria in blood, as evidenced by positive blood cultures ;resence of microbes or their to ins in blood :%o or more o$ the $ollo%in! conditions2 &' Fever (oral temperature *,HQ ) or hypothermia (R,0Q ) (' Deu1ocytosis (*&(6---.D)6 leu1openia (R +6---.D)6 or *&-; bandsO may have a non-in$ectious etiolo!y ,' :achypnea (*(+ breaths.min) +' :achycardiaJ (heart rate *B- beats.min) 5.45 that has a proven or suspected microbial etiology 5epsis with one or more signs of organ dysfunction _ for e ample, '. Cardiovascular, 2rterial systolic blood pressure 6 G8 mm3g or mean arterial pressure J8 mm3g that responds to administration of intravenous fluid ". 4enal, Frine output 68.+ m7)kg per hour for ' h despite ade#uate fluid resuscitation *. 4espiratory, ;aD")K.D" 6 "+8 or, if the lung is the only dysfunctional organ, 6 "88 A. 3ematologic, ;latelet count 6(8,888)7 or +8: decrease in platelet count from highest value recorded over previous * days +. Fne plained metabolic acidosis, 2 p3 6 J.*8 or a base deficit C +.8 m9#)7 and a plasma lactate level C '.+ times upper limit of normal for reporting lab H. 2de#uate fluid resuscitation, ;ulmonary artery wedge pressure C '" mm3g or central venous pressure C ( mm3g 5epsis with hypotension (arterial blood pressure 6G8 mm3g systolic, or A8 mm3g less than patient@s normal blood pressure) for at least ' hr despite ade#uate fluid resuscitation; or &eed for vasopressors to maintain systolic blood pressure G8 mm3g or mean arterial pressure J8 mm3g 5eptic shock that lasts for C' hr and does not respond to fluid or pressor administration $ysfunction of more than one organ, re#uiring intervention to maintain homeostasis

5epsis 5evere sepsis (similar to Nsepsis syndromeN)

5eptic shock

4efractory septic shock ?ultiple-organ dysfunction syndrome (?D$5)

or Pulmonale
H(' ardinal si!n $or the dia!nosis o$ chronic corpulmonale is'. 4aised -ugular venous pressure ". 4ight ventricular failure *. 7eft ventricular hypertrophy A. 4ight ventricular dilatation----------------ans

$iscussion- (3)'J ;-'A+*) Cor pulmonale is right ventricle enlargement4 with or without failure4 secondary to lung parenchyma, pleura or chest wall disease H,' / (B year old unmarried $emale presents %ith dyspnea6 her chest x-ray is normal6 F@ -B(;6 FE@&.F@ -HB; DD )-5B;' )n exercise her oxy!en saturation drops $rom B(; to H0;' #hat is the dia!nosis'. 2lveolar hypoventilation ". ;rimary pulmonary hypertension-----------------ans(%osenta--C &ew drug) *. .nterstitial lung disease A. 2n iety $iscussion- 3)'J ;-'+(8

Pneumonia = Sepsis
H+' >ultiple cysts in lun!s are common in in$ection %ith'. 5taphylococcus----------------ans(?ay 2..?5 "88() ". 5treptococcus *. 9sch. Coli A. 3..nfluenzae $iscussion- 3)'J ;-'H'G 5# pneumoniae is the most common cause of C2; .n ventilator associated pneumonia ?)C non-?$4 pathogen- 5.;neumoniae and ?$4 pathogen- ;. aeruginosa H5' ' NA?-05' score indicate severity $or'. CD;$ ". 2sthma *. ;neumonia-----------------ans A. Ca 7ung $iscussion- ($;/ @8G) 5everity Core adverse features, @CF4%-H+@ score, '. onfusion ". Nrea C Jmmol)7 (J ".( T 'G.H mg :) *. Aespiratory rate C*8)min A. ?; 6 G8 systolic and)or 6 H8 mm 3g diastolic +. 2ge C 05 Score8-' T 3ome treatment possible " T 3ospital !herapy * T .ndicates severe pneumonia

Fun!al In$ection
H0' Ax o$ re$ractory histiocytosis'. Cladarabine----------------------------ans ". 3igh dose ?!R *. 3igh dose cytosine arabinoside A. Kludarabine $iscussion- (2..?5 &ov 8() C7255.K.C2!.D& 2&$ ;2!3D7D/M3istiocytosis'. Class . (3istiocytosis R)7angerhans cell histiocytosis) 9osinophilic granuloma 3and 5hullar Christian disease 7etterer-5iwe disease ". Class ..

*. Class ... !hree classes of childhood histiocytosis are recognised, based on histopathologic findings.!he most wellknown childhood histiocytosis, previously known as histiocytosis R, constitutes class . and includes the clinical entities of Eosinophilic !ranuloma , Hand-Schuller- hristian disease (classic triad of signs consists of diabetes insipidus, e ophthalmos, and bony lesions composed of histiocytes), and DettererSi%e disease. !he name Dan!erhanEs cell histiocytosis(D H) has been applied to the class . histiocytoses. H9' / 5+ year old smo1er man comes %ith $ever6 hemoptysis6 %ei!ht loss and oli!oarthritis' Serial a1ia!ram sho%s $leetin! opacities' #hat is the dia!nosis'. 2%;2------------------ans ". Ca lung *. !% A. BegenerLs granulomatosis $iscussion- (2..?5 &ov 8() /ller!ic bronchopulmonary asper!ilosis (/?P/) , !his results from a 'ype " and """4 hypersensitivity reaction to 2spergillus fumigatus. 9arly on, the allergic response causes bronchoconstriction, but as the inflammation persists, permanent damage occurs, causing bronchiectasis1. !he bronchial asthma of 2%;2 likely involves an .g9-mediated hypersensitivity,whereas the bronchiectasis associated with this disorder is thought to result from a deposition of immune comple es in pro imal airways. Symptoms, Bheeze, cough, sputum(plugs of mucus containing fungal hypae), dyspnoea and recurrent pneumonia4 Investi!ations, CR4(transient segmental collapse or consolidation, bronchiectasis). !he chest roentgenogram may show transient, recurrent infiltrates or may suggest the presence of pro imal bronchiectasis. 3igh resolution chest C! is a sensitive, noninvasive techni#ue for the recognition of pro imal bronchiectasis. 2spergillus in sputum; positive aspergillus skin test and)or aspergillus-specific .g9 425!(radioallergosorbent test); positive serum precipitins; eosinophilia; raised serum .g9 :reatment, &' Prednisolone ". %ronchodilators

:?
HH' #hich o$ the $ollo%in! is a ne% dru! $or /::'. 7inezolid-------------------ans ". !hiacetazone *. Capreomycin A. ;25 $iscussion- 3)'J ;-'88H Second line dru!s !hiacetazone Capreomycin ;25 9thionamide Eanamycin Ciproflo acin Cycloserine 2mikacin 3e%er dru!s 4ifabutin 4ifapentene 7inezolid 2mo )clav combination Dflo acin Clarithromycin 2zithromycin Clofazimine

3e% techniMue(3e% Muestion)- ;-'8'+ .K-gamma release assay (;I) %2C!9C radiometric method,'A-C labelled palmitic acid. .t can detect growth in +-( days. Dnly method to detect live bacilli

HB' :rue about sarcoidosis'. 3ypercalcemia ". 4aise 2C9 *. ?antou negative A. 2ll-----------------------------ans(Evein@s test--C 7ike mantou ) B-' Disseminated tuberculosis is characteriLed by all except'. Kever ". Beight loss *. ?antou test always positive------------------------ans A. ?eninges and adrenals may be involved B&' >rs'8upta is in mid trimester pre!nancy' She has developed :?' ?est treatment is'. 34 ". 349----------------ans(5afety against pyrazinamide is less documented) *. 34] A. 3]9

Pulmonary Embolism
B(' >ost common $eature o$ pulmonary embolism is'. 5weating ". !achypnea--------------ans *. !achycardia A. ;ain chest $iscussion- (3)'J, ; T 'H+') $yspnea is the most fre#uent symptom of ;9, and tachpnea is it@s most freuent sign. B,' / 55 year old lady posted $or hip replacement sur!ery' /ll o$ the $ollo%in! are 1no%n causes o$ D@: except'. ;&3 ". DC; *. .nfections---------------------ans A. 5urgery for ' hour

Pneumoconiosis
B+' ' alci$ication o$ diaphra!matic pleura' is characteristic o$ pneumoconiosis due to'. 2sbestosis----------------ans ". %erylliosis *. 5illicosis A. 2nthracosis $iscussion- 2symptomatic pleural pla#ue B5' In %hich condition6 paradoxical respiration is seen'. 5evere asthma ". %ulbar polio *. $iaphragmatic palsy-------------------ans A. 5ingle rib fracture $iscussion- Klail chest C $iaphragmatic palsy. ;arado ical respirations are seen as movement of the chest wall opposite of what one would e pect to see, i.e. inflation of the lung during e piration. ;arado ical respirations are seen and deflation of the lung during inspiration. !his occurs usually after blunt trauma to the chest, resulting in flail chest (loss of stability of the thoracic cage due to multiple rib fractures, and)or separation of the ribs from the sternum) or with paralysis of the diaphragm.

Dun! ancer
B0' ommonest symptoms o$ primary lun! cancer'. Cough---------------ans(K2I)

". 3emoptysis *. $yspnea A. ;ain chest $iscussion- (3)'J ;T++A) 7ung cancer, o &5C7C((8 :) 5#CC 2denocarcinoma %ronchioalveolar carcinoma o 5C7C("8 :) Dat cell .ntermediate Combined cell type B9' #hich carcinoma o$ lun! responds best to chemotherapy'. 5#uamous cell type ". 2deno carcinoma *. 2naplastic A. 5mall cell type----------------ans $iscussion:reatment2 3on-small cell tumors, 9 cision is the treatment1 of choice. Curative radiotherapy is an alternative if respiratory reserve is poor for 5 . Chemotherapy O radiotherapy for more advanced disease. Chemotherapy ;aclita el O cisplatin Small cell tumors are nearly always disseminated at presentation# !hey may respond to chemotherapy etoposide O cisplatin or etoposide O cisplatinO paclita el or Cyclophosphamide H doxorubicinH vincristine or Cisplatin H radiotherapy &if limited disease). ;leural drainage)pleurodesis for symptomatic pleural effusions 5olitary ;ulmonary &odule with N/round /lassN Dpacity, 2t present, only two radiographic criteria are reliable predictors of the benign nature of an 5;&, lack of growth over a period C" years and certain characteristic patterns of calcification. Calcification alone does not e clude malignancy. 3owever, a dense central nidus, multiple punctate foci, and Nbull's eyeN (granuloma) and Npopcorn ballN (hamartoma) calcifications are all highly suggestive of a benign lesion. &--' >ost common lun! cancer in I3DI/'. 5#uamous ". 2deno-ca *. 5CC7 A. &one-----------------------------ans(5econdaries)

Superior vena cava syndrome- 5uperior vena cava obstruction is most commonly caused by 1bronchogenic carcinoma. .ncreased venous pressure produces edema of the upper body, cyanosis, dilated subcutaneous collateral vessels in the chest, and headache. Cervival lymphadenopathy may also be present. Bhen carcinoma is the cause of superior vena cava syndrome, the treatment is usually palliative and consists of diuretics and radiation.

Pleural e$$usion
BH' JDo% su!ar in pleural e$$usion is a characteristic $eature o$'. CCK ". ;ancreatitis *. !% A. 42-------------------ans BB' ommonest causes o$ pleural e$$usion in India = %orld respectively are'. >iral)>iral------------------ans

". !%)3.> *. !%)C3K A. !%)&ephrotic syndrome due diabetic

5ince answer key was not given answer below are based on e plnations that was provided. >erify them in case of doubt....

E3D) AI3)D)8I
&' /cute $ulminant diabetes is related to'. $E2 ". >iral infection---------------ans *. ?. A. ;regnancy $iscussionFulminant diabetes mellitus (3e% "uestion)2 .t is acute onset diabetes, can occur in any age, occurs after viral infection. 5erum insulin level are reduced but there are no antibodies against insulin or beta cell. $iabetic cheiroarthropathy- 2rthropathy of the -oints of the hands and fingers is a complication of Vuveline onset $?. (' #hich is not a $eature o$ type II D>'. Dbesity ". ;ositive family history *. 7ow serum insulin----------------ans(;/. ?ay 8H) A. .nsulin 4esistance $iscussion/!e at onset ?ody %ei!ht Family h.o Insulin reMuirement Serum insulin level :ype & D> Fsually young ;ean ( thin4 7ess common than in type " 2bsolute (alpha-glucosidase inhibitors can be given) 7ow :ype ( D> 2dults Obese 5trongly Ove D32, later insulin &igh&initially)Dormally or low &late stage)# .nsulin resistance is characteristc .

>)DI 2$ (5trong familial association) Mounger age group &o 2b 1.mpaired insulin secretion (Can be treated by drugs) 3!, 3yperlipidaemia not seen D/D/- Date onset autoimmune diabetes o$ /dults' /2$ antibodies, ;ersons with 72$2 usually test positive for /2$ antibodies, whereas in type ' diabetes these antibodies are more commonly seen in adults rather than in children.

,' 3ot a $eature o$ D7/ is '. !achypnea ". %radycardia----------------ans *. 2bdominal pain A. $ehydration $iscussionDiabetic 7etoacidosis (>.; !opic--C 3)'J ;-""(*)Clinical features-

5ymptoms Dausea :omiting %bdominal pain4 &P " Kune /L) 2ltered mental function 5hortness of breath 5igns-

!achycardia (/II>S 3ov -9) $ry mucous membrane $ehydration 3ypotension @ussmaul respiration4--- d>t metabolic acidosis Kruity odour in breadth--C d)t acetone !achypnea 2bdominal tenderness--C may resemble acute pancreatitis or ruptured viscus Kever may be there (indicates infection)

.nvestigations'. &yperglycemia- %lood sugar A88-H88 mg: ". :D - 7eucocytosis1 . "t is a feature of $@%# "t does not indicate infection &Presence of fever indicate infection4 ---$P 2//.) *. ,ypertriglyceridemia- ,yperlipoproteinemia &dyslipidemia found earliest in $!) A. %F&)Creatinine [ increase (d)t intravascular fluid depletion) +. PseudohyponatremiaJJ (P8I Fune (--9)--C 2 low serum sodium concentration resulting from volume displacement by massive hyperlipidemia or hyperproteinemia, or by hyperglycemia. H. Hyper1alemia($;/)- shifting of EO from intracellular to e tracellular compartment due to decrease in insulin J. ?etabolic acidosis(Eussmaul@s respiration)- 7ow 3CD * with increase anion gap. (2..?5 ?ay "88J) (. ;lasma ketones- ;ositive G. 5. Dsmolality increased, *88-*"8 mosm)kg (&aOE) " O %F&)".( O 4%5)'( &a contributes "J8 of normal "G8 serum osmolality 4%5--C to convert from m.mol divide by '(. 3ere 4%5 increases and hence 5. Dsmolality also increases. '8. 3yperamylasemia ?CI, %lood p3 is the single best test to know the prognosis of a case of $E2 omplications of $E2 Cerebral edema (most dangerous complication)--C 2.;/ "88H. ?ost fre#uently seen in children >enous thrombosis--C 5)C 3eparin given as a 1preventive measure 2dult respiratory distress syndrome ?. 2cute gastric dilatation :reatment'. Fluids --C 8.G: saline ". /nsulin --C 4egular .nsulin I.@ (.nsulin is given I.@ in $E2)--C 2.;/ "88H. 4egular insulin is the only insulin which is given .)> other long acting--C 5)C. 4egular insulin is a short acting insulin. *. 0reatment precipitating e$ents like non-compliance; infection (by antibiotics) A. K1 replacement --C.nitially when patient comes initially he is hyper1alemic, later on when patient is treated with insulin, serum potassium level goes down and may re#uire potassium replacement +. InKection H ), I.@ i$ pH R 9

+' >r' Aam Dal6 H- years6 a 1no%n case o$ diabetes6 %as brou!ht to emer!ency %ard in unconscious state' His blood su!ar %as B-- m!;6 dry ton!ue' ?P H-.5-' Nrine 1etones ne!ative' #hich o$ the $ollo%in! $luid you %ill !ive to treat his mar1ed dehydration'. .n-ecction 47 ". .n-ection &)" saline *. /ive plain water by 4yles tube A. .n-ection & saline------------------------ans $iscussion- (2..?5 &ov "88H) 3)'J ;-""(+ :reatment'. Fluid --C!otal fluid deficit(G-'87) should be reversed over '-" day. .nitially give normal saline to stabiliLe the patients hemodynamically' !hen /ive -'+5; saline 2# =egular "nsulin to be given /23 *. 5ubcutaneous heparin because these patients are prone to venous thrombosis 5' #hich o$ the $ollo%in! is not included in intensive mana!ement o$ diabetes mellitus2 '. ;regnancy ". ;ostural hypotension due to autonomic neurotherapy----------ans *. $? with acute ?. A. ;ost kidney transplant $iscussionDiabetic neuropathy and diabetic $oot care2 (@IP :opic) Dccurs due to ischaemia (absent dorsalis pedis pulses ) and peripheral neuropathy (in-ury or infection over pressure points, eg the metatarsal heads) Si!ns2 3europathy, 7oss of 5ensation($;/), ($ibration is the first to be lost) in gloves S UstockingL distribution, absent an1le -erks ?CI, Callus formation is an one of the early manifestation of diabetic foot :ypes o$ neuropathy in diabetes2 Sensory neuropathy(Commonest), 5ymmetric sensory polyneuropathy, distal numbness(glove and stockingL distribution). $rugs useful for painful diabetic neuropathy--C aspirin )paracetamol , tricyclic (amitriptyline), carbamazepine, lamotrigine; capsaicin cream (a counter-irritant) &ote, &ew drug Duloxitine is being used for painful diabetic neuropathy(;I- ;erspective #uestion) /myotrophy- ;ainful wasting of #uadriceps and other pelvifemoral muscles /utonomic neuropathy (/3P) Features, Postural ,ypotension, 'achycardia, gastroparesis, urine retention (2..?5 ?ay "88(); erection dysfunction (&ov 8(), diarrhea(G+:), Constipation(+:). (&ote, .n diabetic 2&;, $iarrhea occurs in G+: cases, constipation in +: cases). astroparesis, .t may respond to erythromycin. (/II>S >ay -02 It acts on the motilin receptor)' Hypo!lycemia una%areness(/IP8 -0)' Dru! used $or Erectile Dys$unction2 Sildena$il6 /pomorphine6 Iohimbine6 P8-E&(/II>S 3ov (--H)' *rd &erve may be involved but pupillary reaction is normal. 0' #hich electrolyte imbalance can occur in a diabetic patient %ith normal blood urea = serum creatinine'. 3yperkalemia-----------------ans(4!2 !ype A can cause 3yperkalemia) ". 3ypokalemia *. 3ypernatremia A. 3ypocalcemia 9' Epalrestat is a'. 7ipid lowering agent ". 2nti hypertensive drug

*. 2nti diabetic drug--------------ans A. &one $iscussion- (&ew Iuestion) Pathophysiolo!y of diabetic complication .n diabetes glucose is converted into sorbitol which is a tissue toxin4 9nzyme re#uired in this step is aldose reductase4& lucose -----aldolase reductase------ 5orbitol) 4ecently a new drug 4palrestat 5678 has been launched which is a aldose reductase inhibitor (&I) which is supposed to reduce the complication of diabetes. H' #hich o$ the $ollo%in! is a $ood o$ hi!hest !lycemic index'. %utter ". ;otato *. 4ice A. 4asgulla----------------ans $iscussion- (2.;/ "88J) 2void food of high glycemic inde (i.e. any food like sugar, glucose powder etc.which are absorbed immediately in /.! and raise blood sugar very fast). D*7 Cornflake is a food of very high glycemic index4 %esides glucose and water, alcohol is absorbed immediately in /.!. %ut, when alcohol O fatty food--C &asha der se aayega. B' :rue about Sama!yi phenomena *am (am '. 3ypoglycemia 3ypoglycemia ". 3ypoglycemia 3yperglycemia *. 3yperglycemia 3yperglycemia A. 3yperglycemia 3ypoglycemia $iscussion- 2nswer-" (2..?5 &ov "88+) Somo!yi phenomena- .t is the morning(( am) hyperglycemia which follows hypoglycemia at late night (" am). !his occurs due to release of counter regulatory hormones (glucagon,'st to be released, followed by adrenaline--C epinephrine not other Catecholamine, steroids, growth hormones thyroid). !reatment is to reduce the night dose of insulin &-' Detemir is a'. 5ulphonylurea ". .nsulin--------------------ans *. !hiopglitazone A. 2lpha glucosidase inhibitor $iscussionPharmacolo!y o$ Insulin'. Fltra short acting, 7ispro, 2pert1, /lulisine1 ". 5hort acting, 4egular insulin *. .ntermediate acting, 7ente, &;3 A. 7ong acting, Fltra 7ente, ;]., "nsulin largine , Determir (3e% Muestion) +. .nhalation insulin, 9 ubera(&ew drug)--C Bithdrawn (7ung Ca increases) > "2 8lulisine6 /spart6 8lar!ine6 Detemir6 and Exubera are ne% insulin &&' #hich o$ the $ollo%in! is a 8DP-& a!onist '. 5itagliptin ". /lulisine *. 9 enatide--------------ans A. &one $iscussion3e% concepts and ne% oral dru!s in diabetes2

&' /mylin e$$ect(Physiolo!y)2 '. &ormally when we eat food, intestine secretes amylin (released by beta-cells of pancreas with insulin--C both deficient in diabetes ". 2mylin reduces glucagon secretion and slows down the gastric emptying ,' Pharmacolo!y2 &ew anti diabetic drugs which increases amylin secretion(2mylin agonist)--C Pramlintide (' Incretin e$$ect (Physiolo!y)2 '. &ormally when anybody eats food it causes increased insulin secretion ". .n a new study it is seen that normal people who take oral glucose tend to have higher serum insulin level as compared to those normal people to whom glucose was given intravenous although blood sugar was same in both the groups of people. .t is because oral glucose causes release of some polypeptide from the intestine which raise serum insulin level (Insulinotropic polypeptide). *. !hese polypeptide include 8DP-&6 8IP(5ecreted by 7-Cells of intestine) A. !hese two polypeptide are degraded by DPP-I@ EnLyme +. Pharmacolo!y, &' 3e% anti diabetic dru!s %hich raised 8DP-& and 8IP (in-ection)'. 9 enatide1 ". 7iraglutide (' 3e% antidiabetic dru!s %hich raised 8DP& and 8IP level by inhibitin! DPPI@(DPP-I@ inhibitors)--C Dral and hence very popular '. 5itagliptin ". >ildagliptin &ote, 2nti-diabetic drugs which do not cause hypoglycemia'. ?etformin ". 2lpha glucosidase inhibitor *. !hioglitazone A. $;;-.> inhibitor &(' /s per F3 @II criteria tar!et !oal o$ ?P control in diabetes patients is'. 6'*8)(8----------------ans ". 6'"+)(8 *. 6'"8)(8 A. 6'"+)J+ $iscussion- %; 6 '*8) 6(8 mm3g (6'"+ ) 6J+ is associated with renal disease) &,' / 9- years old patient %as brou!ht to emer!ency %ard in a comatose state' His /?8 %as done and $indin!s are as $ollo%s3a G &,5 meM.lit6 7 G 5 meM.lit H ), G &- meM.lit ?lood !lucose G (- mmol.lit ?N3 G + mmol.lit (3ormal value o$ ?N3 G ('H0 -9'&+ mmol.D or H to (- m!.dD) #hat is your dia!nosis'. &on ketonic hyperosmolar diabetic coma ". $E2 *. 3ypoglycemic coma A. 7actic acidosis $iscussion- Confusing #uestion(2.;/ "88J) "n this case patient has metabolic acidosis because his serum bicarbonate is 1/meA>lit# 5o possibility of choice % is ruled out# ,is blood sugar is 2/ mmol>lit i#e# 3L/ mgJ &2/ x1MG3L/) so choice C is ruled out *oth choice 2 and . are possible in this case but patient8s age is E/ yrs so better ans is .N##$r *hatia###"f age was 1E then--- $@%#############

" think the Auestion was about the student?s ability to differnentiate b>n $@% and ,,5########and clearly answer is $@%###also bicarbonate level is normal to slightly decreased in ,,5###which also supports the answer#######&,>1E P-22M2) &+' Pretibial myxedema is seen in'. 3yperthyroid ". 3ypothyroid *. /rave disease---------------------ans A. 3ashimoto thyroiditis $iscussion- 3)'J ;-""** (!able **+-H) /raves 3yperthyroidism $ermatopathy--C !ibial my edema Dphtalmopathy Extra Muestion2 /$ is seen in'. 3ypertheroidism-----------------ans ". 3ypotheroidism *. /rave disease A. 3ashimoto thyroiditis &5' / +- year old %oman present %ith a $irm nodular midline nec1 mass' ?lood tests reveal the presence o$ antibodies to thyro!lobulin' Di1ely cause is'. /ravesL disease ". !hyroid carcinoma *. !o ic multinodular goitre A. 3ashimotoLs thyroiditis-----------------ans $iscussionHashimotoEs thyroiditis ?ost common auto-immune cause of goitrous hypothyroidism "8-H8 yrs old women /nti thyro!lobulin antibodies and :hyroid peroxidase antibodies are present in the serum 2ntinuclear factor (2&K) may be positive !hyro ine therapy &0' /ll are true about de "uervainEs thyroiditis except'. 4aised 954 ". 5ubsides spontaneously *. 2utoimmune etiology---------------------------ans A. ;ain and swelling of the thyroid occurs $iscussion- >iral infection (%O7 "+)e ;-(88) &9' Pituitary apoplexy is not associated %ith %hich o$ the $ollo%in! medical conditions'. $? ". 3yperthyroidism---------------ans *. 5ickle cell anaemia A. 3ypertension $iscussion- (2.;/ 8() 4ef. 3)'J ;-"'G( Extra Muestion(/II>S >ay -9)2 First hormone to decrease a$ter Pituitary radiation is'. /3-----------ans ". !53 *. 2C!3 A. K53 $iscussion- !hough the pattern of hormone loss is variable, after cranial irradiation, /3 deficiency is most common, followed by gonadotropin !53 and 2C!3 deficiency

&B' Secondary hyperparathyroidism is seen in all except'. Chronic renal failure ". CushingLs syndrome-------------------------ans *. ?alabsorption of calcium from gut A. 4ickets $iscussion- (2. 8H) P:H Primary Secondary :ertiary I' .ncrease .ncrease .ncrease a44 .ncrease Decrease .ncrease

Primary hyperparathyroidism auses2 !he most common cause of primary hyperparathyroidism is parathyroid adenoma . .mportant point, 2denomas are most often located in the inferior parathyroid gland (?CI). Chief cells are predominant in both hyperplasia and adenoma

... Secondary hyperparathyroidism - Ca91 decrease, ;!3 .ncrease (?CI) auses, 7ow vitamin $ intake, C4K, 4ickets, Dsteomalacia, ?alabsorption .... :ertiary hyperparathyroidism- Ca"O .ncrease, ;!3 .ncrease (K2I)
auses, Dccurs after prolonged secondary hyperparathyroidism, causing glands to act autonomously having undergone hyperplastic or adenomatous change. !his causes increase in a(4 from secretion of ;!3 unlimited by feedback control . 5een in chronic renal failure.

I@' >ali!nant hyperparathyroidism auses, Parathyroid-ralated protein (;!3rp) is produced by some squamous cell lung cancers, breast and renal cell carcinomas. (2..?5 ?ay "88J) &ote, 3ypercalcemia in malignant occur due to increase level of P',rp. .n this ;!3 level are not raised (7I) &H' Hypercalcemia is seen in all except'. ;rolonged immobilization ". ?ultiple myeloma *. 5arcoidosis A. 2cute renal failure-------------ans $iscussion- (2..?5 &ov 8J) '. )steoblast- !hese are the bone forming cell. Bhenever there is increase activity of osteoblast it causes raised level of serum al1aline phosphatase (?CI). !hat is why level of serum alkaline phosphatase is raised in children and pregnancy (?CI) ". )steocytes- !hey are the bone maintaining cells. !hey regulate bone activity. *. )steoclast- !hey cause bone resorption(?CI). .n ?ultiple myeloma there is increase activity of osteoclast that is why there are lytic lesion and Hypercalcemia in ??(?CI) 3ote, .n >> there is no increase in activity of osteoblast. !hatLs why serum alkaline phosphatase level are normal in ?? and bone scan is also normal )ther causes o$ Hypercalcemia2 '. 7ithium, !hiazide (Furosemide--C Ca loss) ". Ca of breast, 7ung, kidney, ??, lymphoma *. >it $ into itation, >it 2 into ication, 2luminium into ication A. 3yperparathyroidism, 3yperthyroid +. 5arcoidosis H. ;rolong immobilization J. ?ilk alkali syndrome (-' #hich is not true o$ hypocalcemia-

'. Can occur in tumor lysis syndrome ". .nverse relation with ?gOO levels-----------------ans *. ;rolonged I! interval A. 7atent tetany is seen $iscussion- Dption "--C .nverse relation is only found in C4K and /ietelman@s syndrome 7atent tetany Chvostek sign !rousseau sign 3ypocalcemia--C Causes, '. .diopathic (Autoimmune )- associated with other autoimmune disorders. 7ab tests of idiopathic hypoparathyroid [ Ca"O (decrease), ;DA (.ncrease) ". .nfantile hypoparathyroidism, .t is associated with thymic aplasia (Di-:eorge syndrome ) *. ;ost operative, 5urgery (thyroidectomy)--C 7eads to transient hypocalcemia A. Pseudohypoparathyroidism(Ca decrease, ;!3 .ncrease) is a group of disorders characterized by hypocalcemia due to renal resistance to ;!3. !here are several subtypes caused by different mutaions involving the ;!3 receptor or it@s / protein or adenyl cyclase. ;!3 levels are high and the ;!3 receptors in bone are typically involved, such that bony changes of hyperparathyroidism may be evident. >arious phenotypic abnormalities may be associated [ classically, short stature, round face, obesity, short $ourth metacarpals, ectopic bone formation, and mental retardation (&' ushin!Es disease is characteriLed by'. .ncreased urinary catecholamines ". .ncreased serum 2C!3 and serum cortisol-------------------ans *. .ncreased serum 2$3 A. $ecreased serum 2C!3 and increased serum cortisol $iscussion- !raditionally, only an individual who has an 2C!3-producing pituitary tumor is defined as having Cushing@s disease, whereas Cushing(s syndrome refers to all causes of excess cortisol7 exogenous %C', tumor, adrenal tumor, pituitary %C',-secreting tumor, or excessive glucocorticoid treatment# ushin! Syndrome 2) / :H-dependent causes, (.ncrease 2C!3) '. ushin!Es disease2 %ilateral adrenal hyperplasia due to an / :H secretin! pituitary adenoma ". Ectopic 2C!3 production, 9specially smell cell lung cancer and carcinoid tumours&!CB), ;ancreatic Ca, bronchial adenoma %) / :H-independent causes, (decrease 2C!3 due to [ve feedback) '. Iatro!enic, ;harmacological doses of steroids(common) ". /drenal adenoma or carcinoma , may be associated with abdominal pain and virilization in women *. /drenal nodular hyperplasia auses o$ ushin! Syndrome .atrogenic (drug--C5teroid) ;ituitary adenoma (Cushing@s d)e) 9ctopic 2C!3 secreting tumor 2drenal adenoma)Ca S' / :H 4educed 4aised 4aised 4educed Dia!nosis 3istory C!(3ead) C!(Chest) C!(abdomen) Ax 4educe dose 5urgery 5urgery 5urgery

Features o$ ushin!Es syndrome2 &' Symptoms due to excess o$ !lucocorticoid '. !he loss of proteins from the muscle in particular causes se$ere -eakness of muscles (/II>S >ay -9) ". !he protein collagen fibres in the subcutaneous tissue are diminished so that the subcutaneous tissue tear easily resulting in development of large purplish striae where they have torn apart. *. 9levated blood glucose concentration

A. ?obilization of fat from the lower part of the body with concomitant e tra deposition of
fat in the thoracic and upper abdominal regions giving rise to a buffalo torso. !he e cess secretion of steroid also leads to an edematous appearance of the face (Y moon faciesZ). (' Symptoms due to excess o$ mineralocorticoid '. 5alt and water retention leads to hypertension ". 5ignificant E O depletion(,ypokalemia)

,' Symptoms due to excess o$ andro!en'. ". *. A. 3irsutism Kacial acne Dligomenorrhea 2menorrhea

((' /ll o$ the $ollo%in! %ill cause myopathy except'. Cushing syndrome ". R-linked hypophosphatemic rickets-----------------ans(2..?5 ?ay 8J) *. &utritional osteomalacia A. Dncogenic osteomalacia (,' /ll are $eatures o$ primary hyperaldosteronism except'. 3ypokalemia ". 3ypertension *. ?etabolic acidosis-----------------ans A. 7ow plasma rennin $iscussiononn's Syndrome ;rimary hyperaldosteronism d)t aldosterone secreting adrenal adenoma 9dema is not a feature of ;rimary aldosteronism but a feature of 5econdary aldosteronism Epleronone- 2ldosterone receptor antagonist (+' /ddisonEs disesase is characteriLed by'. 3ypokalemia ". ;eptic ulceration *. 3ypoglycemia------------------ans A. .ncrease aldosterone $iscussion- (2..?5 8J) /DDIS)3ES DISE/SE (3ypofunctioning of adrenal corte ) .ncrease 2C!3--C %ronze ;igmentation of skin ?ineralocorticoid deficiency 3yponatremia with contraction of 9CK >olume 3yperkalemia 2cidosis /lucocorticoid deficiency--C 3ypoglycemia (5' FreMuency o$ tunin! $or1 used to test vibration sense is '. '"(----------------ans ". "+H *. +'" A. '8"A $iscussion- Kor checking posterior column involvement --C do 4homberg@s test ;luoripotent stem cells'. 7 '. %--C C77 ". !--C ?ycosis fungoides

". ? '. 4%Cs --C ;> ". ;latelets --C 9! *. &)%)9)? --C C?7--C ?yeloproliferative disorders

3EPHA)D)8I
5,' )smotic demyelination syndrome occur in treatment o$'. 2cute /& ". 5.2$3-----------------ans *. C4K A. F!. $iscussion- (K2I) 3yponatremia (sodium6'*+m9#)7) 7ow serum osmolality %F& and serum uric acid tends to fall because of plasma dilution 5erum potassium and %icarbonate levels are normal in 5.2$3 :.t o$ SI/DH2 2cute 5.2$3 '. 5tandard first line therapy is -ater restriction, then ". .nfusion of hypertonic saline (*:) 2 rapid correction will produce central pontine myelinolysis (Dsmotic demyelinating syndrome) which is an acute potentially fatal neurological syndrome characterized by #uadriparesis, ata ia and abnormal e traocular movements hronic SI/DH2 3yponatremia can be corrected with-C Demeclocycline or fludrocortisone 5+' Pseudohyponatremia occurs in %hich o$ the $ollo%in! conditions'. 5.2$3 ". C3K *. 3yperlipidemia-----------------ans A. 5evere dehydration $iscussion, 2n errorneously low serum sodium concentration d)t volume displcement by massive hyperlipidemia)hyperproteinemia)hyperglycemia 55' /ll occurs in mali!nant hyperthermia except'. 3yperkalemia ". %radycardia---------------ans *. 3ypertension A. $.>C $iscussion- (2..?5 ?ay "88J) ?alignant 3ypethermia is clinical syndrome observed during general anaesthesia associated with rapidly increasing temperature as great as '1C)+min. .t has mutations in the ryanodine receptor. Clinical features of which has been described as, 3yperthermia (temperature may rise to more than '8+1K). !he heat production is due to increased muscle metabolism(both aerobic and anaerobic), glycolysis and hydrolysis of high energy phosphates involved in the process of contraction)rela ation 9tCD" may rise more than '88mm 3g (normal *" to A" mm 3g).!his is the most sensitive early sign of malignant hyperthermia !2C3MC24$.2 (and not %42$MC24$.2). 3ypertension and Cardiac arrhythmias can also occur 5evere metabolic acidosis (p3 6 J.8) 3yperkalemia, muscle rigidity, increased rigidity, increased creatine phosphokinase, increased myoglobin

4enal failure, $.C, pulmonary and cerebral edema $eath 2cidosis- 3yperkalemia 2lkalosis- 3ypokalemia

50' Interpret the $ollo%in! data H ), G H meM.lit pH G 9'+P )( G (- mmH! P)( G B- mmH! 3a G &,0 meM.lit' 7 G + meM.lit #hat is your dia!nosis'. Fncompensated metabolic acidosis ". Fncompensated 4espiratory acidosis *. Compensated 4espiratory acidosis A. Compensated metabolic acidosis-----------------ans $iscussion- 5ince all options given are acidosis means p3 should have been lower than J.*+ but it is J.A here means it is compensated, and acidosis is d)t decrease in bicarbonate level as it is here, so metabolic acidosis component is primary, which has been compensated by resp. alkalosis as decrease in CD" suggests; to bring p3 to normal. ?asic $undamental 2) 2cidosis accur by two mechanism '. /ain of acid ". 7oss of alkali %) 2lkalosis occur by two mechanism '. 7oss of acid ". /ain of alkali 3ote2

a) CD" is an acid, being controlled by lungs. 3CD* is an alkali, being controlled by kidneys. b) 2ny disturbance of CD" leads to respiratory acidosis)alkalosis c) 2ny disturbance of 3CD* leads to metabolic acidosis)alkalosis
59' Interpret the $ollo%in! data H ), G HmeM.lit Ph G 9'(p )( G H- mm H! p)( G B- mmH! '. ?i metabolic acidosis S respiratory acidosis------------------ans ". ?i metabolic acidosis S respiratory acidosis *. ?i metabolic acidosis S respiratory alkalosis A. ?i metabolic alkalosis S respiratory alkalosis $iscussionDisturbance Aespiratory acidosis Aespiratory al1alosis >etabolic acidosis >etabolic al1alosis Primary alteration .ncreased pCD" $ecreased pCD" $ecreased plasma 3CD* .ncreased plasma 3CD*

5H' /ll o$ the $ollo%in! can cause hi!h anion !ap acidosis E< EP:'. 9thylene glycol ". 5tarvation *. /lue-sniffing----------------------ans

A. $iabetic ketoacidosis $iscussion- (2..?5 ?ay "88() 3)'J ;-"G8 >E:/?)DI / ID)SIS-

'. 3ormal anion !ap metabolic acidosis- /lue sniffing)4!2 ". Increased anion !ap metabolic acidosisa) "ncreased unmeasured anions4
'. 7actic acidosis ". Eetoacidosis--C 2lcoholic)$iabetic)5tarvation b) Poisoning '. 5alicylates ". ?ethanol *. 9thylene glycol c) %ccumulation of endogenous acid4 --- 24K and C4K /3I)3 8/P, W3a P ( l 4 H ),)S Fnmeasured anions in the body are anionic proteins, phosphates, sulphate and organic anions. Fnmeasured cations in the body include calcium, magnesium and potassium (not sodium) .n e#uilibrium state, &ormal plasma anion gap T 'A O A (?CI) 5B' In %hich type o$ Aenal tubular acidosis6 hyper1alemia occurs'. !ype .>----------------------ans ". ;ro imal 4!2 *. $istal 4!2 A. 5een in all above cases $iscussionAenal tubular disease (2ll #uestions come from this chart only) !ype .> !ype .. (;ro imal) !ype . 0-' Fusion o$ $oot process occurs in'. ?;/& ". ?embranous /& *. K S 5/----------------------ans A. &one $iscussion- Chart 4eview 4D2?5 ;-'G( 0&' Aamlal6 a 05 year male6 a 1no%n case o$ a colon complains o$ !eneraliLed s%ellin! in his body' His urine examination sho%s proteinuria o$ + !m.(+ hrs' #hich type o$ 8lomerulonephritis is most li1ely to be caused in this'. ?inimal charge ". ?embranous-------------------ans *. Kocal S segmental A. ?embranous proliferative 0(' omplement level are reduced in all except'. ?embranoproliferative /& ". 579 *. ;ost streptococcal /&--------------------ans A. Cryoglobulinemia $iscussion- (Controversial Iuestion, 7I-7atest Iuestion) 5een in all but returns to normal in *(Ck in #uestion if it should be increased in place of reduced from 2..?5 paper) .ncreased E $ecreased E, Fanconi syndrome $ecreased E, Frine p3 C +.+, 4enal stone

0+' Aani6 a (+ year lady6 !ave birth to a healthy child6 + days later6 she complaints o$ inability to pass urine6 she %as dia!nosed as havin! /AF = %as treated by repeated dialysis' Even & year a$ter that also she could not re!ain her normal renal $unction' #hat could had been her problem'. ?embranoproliferative /& ". ?esangioproliferative /& *. 2cute cortical necrosis-------------------------ans A. Kocal S segmental glomerulosclerosis $iscussion- 4enal cortical necrosis--C emedicine(classical presentation) suppported by one more source...others fairly ruled out('and " are same thing; A presents as nephrotic syndrome) 05' #hich is not a $eature o$ ?arter syndrome'. 3!------------------------ans ". $ecreased EO *. ?etabolic alkalosis A. 2ll $iscussion- %/7/ ?artterJ 8itelman Diddle 8ordon 7 $ecreased $ecreased $ecreased IncreasedJ >etabolic 2lkalosis 2lkalosis 2lkalosis /cidosisJ ?P & & ,igh &igh S' /ldosterone .ncreased .ncreased $ecreased Decreased

?assive bleeding, C'888 ml blood loss /ive blood for blood (5aline won@t do.....)

00' Features o$ 8ordonEs syndrome (Pseudo hypo aldosteronism)2 '. 3! ". .ncreased EO *. ?etabolic acidosis A. 2ll-----------------------ans 09' /cute tubular necrosis (/:3) is characteriLed by'. 3yperkalemia ". $ilutional hyponatremia *. ?etabolic acidosis A. 2ll-------------------------------ans $iscussion- 2!& presents with acute renal failure (24K) and is one of the most common causes of 24K. !he presence of Nmuddy bro%n castsN of epithelial cells found in the urine during urinalysis is pathognomonic for 2!&. 0B' /ll improves a$ter dialysis except'. ;ericarditis ". ;eripheral neuropathy--------------ans *. ?etabolic acidosis A. 5eizure $iscussion- 4ef. Biki--C $ialysis (confirm again) 9&' Emphysematous pyelonephritis is caused by'. 9.coli---------------ans ". ;roteus *. ;seudomonas A. Elebsiella $iscussion- a- 3)'J- '("H Dccurs in diabetic patients, often in concert with urinary obstruction and chronic infection. 9mphysematous pyelonephritis is usually characterized by a rapidly progressive clinical course, with high fever, leukocytosis, renal parenchymal necrosis, and accumulation of fermentative gases in the kidney and perinephric tissues. ?ost patients also have pyuria and glucosuria. E' coli causes most cases, but occasionally other 9nterobacteriaceae are isolated.

9,' hronic tubulo interstitial disease is caused by'. 7ithium ". Cyclosporin *. 3yper uricemia A. 2ll------------------------ans $iscussion- 3)'J for detailed list auses o$ :ubulointerstitial Disease/' :oxins &' Exo!enous toxins'. 2nalgesic nephropathy ". 7ead nephropathy *. ?iscellaneous nephroto ins (antibiotics, closporine, radiographic contrast media, heavy metals) (' >etabolic toxins '. 2cute uric acid nephropathy ". /outy nephropathy *. 3yperkalemic nephropathy A. 3ypokalemic nephropathy +. ?iscellaneous (hypero aluria, cystinosis) ?' 3eoplasia '. 7ymphoma ". 7eukemia *. ?ultiple myeloma 9+' #hich o$ the $ollo%in! statements about adult polycystic 1idney disease is true'. 2utosomal recessive disorder ". 7ow erythropoietin level *. 3ematuria can occur----------------------ans A. %erry aneurysm donLt have association with polycystic kidney disease $iscssion/utosomal dominant polycystic 1idney disease(/DP7D)/enes on chromosomes 'H (;E$') and A (;E$") ommon linical Features'. Klank pain and vague abdominal discomfort ". 2cute loin pain or renal colic due to hemorrhage into the cysts *. ,ypertension may appear after age "8 years A. &octuria, hematuria and urinary infection appear in the third and fourth decade +. Fraemia H. 3igh erythropoietin level(read from google books and multiple sites on internet) J. &ephrolithiasis commonly due to calcium oxalate may be there M# Cysts may be there in liver, pancreas, and spleen#Cysts are not present in lungs&extra-abdominal) G. "ntracranial &berry) aneurysms also may be present which can cause 523 '8. 2zotemia is usually progressive ''. Commonest e tra renal manifestation is colonic diverticulosis '". $iagnosis is by .>; or ultrasound 99' )li!uria denotes urinary output less than'. A88ml ". '88ml *. "88ml A. +88ml-----------------------ans $iscussion- 2nuria TC 6 '88 ml /AF-

'. Dliguric 6 +88 ml)day (2nuric is called when 6 '88) ". &on-oliguric C +88 ml)day (3ere urine is of poor #uality i.e. contains little waste because blood is not filtered well, despite the fact that an ade#uate amount of urine is e creted ( C+88 ml) 9H' 3ormal urine contains protein secreated by tubules6 %hich is that'. .g2 ". !om-3orsfall *. Frokinase A. 2ll-------------------------ans $iscussion- Commercially urokinase comes from urine only

8I:4DI@EA
H-' /n &Hyrs old male presents %ith haematemesis6 he has history o$ mar1ed $ever $or the past &+ days $or %hich he %as mana!ed %ith dru!s6 moderate splenome!aly is presentO most li1ely dia!nosis'. $uodenal ulcer ". $rug induced gastritis--------ans *. 9sophageal varices A. &one of the above $iscussion- Kactor against diagnosis of portal hypertension--C no ascites, age of presentation (ref-harrison, ;T'GJJ, chap-*8") H, 'Initial :reatment o$ choice $or Collin!er Ellison syndrome is'. Dctreotide ". /astrectomy *. 3" blocker A. ;roton pump inhibitor-----------------ans $iscussionCollin!er-Ellison syndrome 2 non-beta islet cell tumor that produces !astrin and is associated with gastric acid hypersecretion and peptic ulcer disease !he tumors are biologically malignant in H8: of cases !umor size varies from " mm to "8 cm :umor distribution ?a-ority of gastrinomas occurred within the pancreas linical >ani$estations2 /) Peptic ulcer %) Diarrhoea C) Presence o$ >E3 I (;-A"G 4D2?5) Dia!nosis2 '. Fastin! !astrin level' Kasting gastrin levels are usually 6'+8pg)ml. >irtually all gastrinoma patients have a gastrin level C '+8-"88pg)ml ". 2 %2D)?2D ratio C8.H is highly suggestive of ]95, but a ratio 68.H does not e clude the diagnosis %2D- %asal 2cid Dutput ?2D- ?ean 2cid Dutput ,' 8astrin provocative tests 5ecretin in-ection test- 2 parado ical (3)'J ;-'(HG) increase in gastrin of C "88 pg within '+ min of secretin in-ection has a sensitivity and specificity of CG8: for ]95 Calcium infusion test- less sensitive and specific than secretin test A. :umor DocaliLation 9ndoscopic ultrasound (9F5) permits imaging of the pancreas with a high degree of resolution (6+mm)


Ax2

5omatostatin analogue '''-.n-pentetreotide (Dctreoscan) with sensitivity and specificity rate of C J+:. Kor metastases, 2bdominal C! scan, ?4., or Dctreoscan

;;.s (Dmeprazole) are the treatment of choice 5urgery--C resection of tumor in pancreas (earlier total gastrectomy was primary operation)--C /!-(J I-HG

:herapy o$ metastatic endocrinea. 5treptozotocin b. +-fluorouracil c. $o orubicin d. .K&-alpha e. 3epatic artery embolization f. '''-.n-pentetreotide Secretin- .t is produced in the 5 cells of the duodenum in the crypts of 7ieberk`hn. .t stimulates the secretion of bile from the liver. .t also increases watery bicarbonate solution from pancreatic duct epithelium. 5ecretin increases water and bicarbonate secretion from duodenal %runner@s glands in order to buffer the incoming protons of the acidic chyme. .t also enhances the effects of cholecystokinin to induce the secretion of digestive enzymes and bile from pancreas and gallbladder, respectively. .t counteracts blood glucose concentration spikes by triggering increased insulin release from pancreas, following oral glucose intake. .t also reduces acid secretion from the stomach by inhibitin! !astrin release from / cells. .n addition, secretin stimulates pepsin secretion, which can help break down proteins in food digestion. H+' / +- year old obese lady comes to you %ith Kaundice' Her DF: report is as $ollo%s?ilirubin2 &- m!; Direct2 0 m!; S8):2 &( m!; S8P:2 &+ m!; /l1aline phosphatase2 &+ Serum Protein2 0 !m; Serum /lbumin2 ( !m; Serum 8lobulin2 + !m; >ost li1ely dia!nosis'. 2cute hepatitis ". Chronic liver disease-------------------ans(was written in classroom but ck from other sources) *. /all stone A. $ubin Vohnson syndrome $iscussion- 4eversal of 2,/ ratio HH' Ax o$ hepatitis ?'. 7amivudine ". .K *. %oth -----------------ans A. &one $iscussionDru!s used $or treatment o$ hepatitis ? are2 '. .nterferon alpha ". ;ergylated .nterferon 3# ;amivudine A. 2defovir dipivo il +. 9ntecavir (an oral guanosine analogue polymerase inhibitor)

3e%er dru!s $or Hepatitis ? '. !elbivudine ". 9mtricitabine *. !enofovir A. Clevudine !reatment or hepatitis C--C .nterferon alpha O 4ibavirin (oral guanosine nucleoside)

B(' In spider neavi %hich o$ $ollo%in! hormone is responsible $or dialation o$ arteries'. 9strogen-----------------ans ". $3925 *. !estosterone A. 3epato !o in $iscussion- (2..?5 &ov 8() :reatment'. ,entoxyphylline in alcoholic hepatitis , .n severe alcoholic hepatitis, oral pento ifylline reduces mortality particularly from hepatorenal failure (;ento yphylline is an analogue of theophylline it is used to treat intermittent claudication) ". 8lucocorticoids is helpful in pts with severe alcoholic hepatitis. %ut no role in established cirrhosis B,' Hepatic-encephalopathy may be precipitated by all o$ the $ollo%in! except'. /. bleed ". %arbiturates *. 3yper kalemia----------------ans A. 5urgery $iscussion- (2.;/ 8J) /cidosis !ood--* Hyper1alemia !ood Precipitatin! $actors2 I' Increased nitro!en load2 " bleeding (most common precipitating factor) 9 cess dietary protein 2zotemia Constipation II' Electrolyte and metabolic abnormality2 ,ypokalemia, %lkalosis, 3ypovolumia, 3yponatremia III' Dru!s2 5edatives, Excess diuretics I@' Stress, .nfection, 5urgery B+' #hich is not a $eature o$ non-cirrhotic portal H:'. 5plenomegaly ". 3ematemesis *. 2scites-----------------------ans A. 2ll are seen $iscussion- (K2I) 3)3 IAAH):I P)A:/D FI?A)SIS , .t is also called Idiopathic portal hypertension. .ts incidence is very high in .ndia accounts for '+: to '(: of all patients with portal hypertension linical $eatures2 "assi$e hematemesis )plenomegaly is present 4arely patient develop encephalopathy

%scites is uncommon Vaundice and signs of liver cell failure are uncommon

Investi!ations2 &ormal liver function test Dormal P' 5. ;roteins normal ;ortographyo 9eeping willow appearance [ sudden cut off peripheral portal vein branches o 'ree in winter appearance [ filling of a large number of collaterals with gross distortion of intrahepatic pattern o 5plenic vein is dilated and tortuous :reatment- .n the lines of usual variceal bleed B0' False about hemochromatosis'. 3ypogonadism ". 3epatoma can occur *. ;seudo /out can occur A. $esferrio amine is treatment of choice------------------ans(>enesection) $iscussion- (2.;/9 "88() 3emochromatosis causes bronze diabetes. >ana!ement- ?ain treatment consists of weekly venesection of +88 ml blood. !he chelating agent defero amine (.)> or 5)C) is indicated for patients with hemochromatosis or in those with secondary iron overload due to thalassemia who cannot tolerate phlebotomies. 2n oral chelator--C De$erasirox is new drug B9' #hich o$ the $ollo%in! is implicated in patho!enesis o$ rohnEs disease'. ?ycobacterium avium paratuberculosis----------------ans ". ?ycoplasma *. 4ota virus A. 5treptococci $iscussion- (2..?5 ?ay 8() C$ .nfectious etiologies&' ?acterial 5almonella 5higella !o igenic 9scherichia Coli Campylobacter Mersinia Clostridium difficile /onorrhea Chlamydia trachomatis (' >ycobacterial--* ?. avium ,' Parasitic 2mebiasis .sospora !richuris trichiura 3ookworm 5trongyloides +' @iral Cytomegalovirus 3erpes simple 3.> 5' Fun!al 3istoplasmosis Candida

2spergillus

2therosclerosis--C Chlamydia "' / +- yr old ci!arette smo1er complains o$ epi!astric pain6 %ell localised non radiatin! and described as burnin!' :he pain is partially relieved by eatin!' :here is no %ei!ht loss' He has not used 3S/ID' :he pain has !radually %orsened over several months' :he most sensitive %ay to ma1e a speci$ic dia!nosis is'. %arium -ray ". 9ndoscopy *. 9C/ A. 5erum gastrin $iscussion- (/!-(J I-'HH) 7ocalised epigastric burning pain relieved by eating rer#uires evaluation of peptic ulcer disease. Fpper /. endoscopy provides the best sensitivity and specificity; barium swallow is less e pensive, but is less accurate in defining mucosal disease. ;atients with refractory or recurrent disease should have serum gastrin levels measured to rule out gastrinoma. 2 positive antibody test for 3.pylori would only indicate previous e posure. "-((' :he earliest mani$estations o$ serious !ram Pve in$ection may consist o$ a triad o$ si!ns that include'. !achypnoea, hypotension, and an altered sensorium ". !achypnoea, hypotension and lactic acidosis *. !hrombocytopenia,hypotension and lactic acidosis A. ?ild hyperventilation, respiratory alkalosis and an altered sensorium 2nswer-A (ck with medicine) 5.45amay have non infectious etiology '. !emperature 6*H or C*( degree centigrade ". 34 C (8 bpm *. !7C C'"888 or 6A888 A. 44 C"8, ;aCD" C*" +. 5epsisa 5.45 O $ocumented blood infection 5epticemia(5epsis 5yndromea ck differs from %hatia notes)a 59;5.5 O Dne or more organ dysfunction Classical triada 3yperventilation, respiratory alkalosis and an altered sensorium Kor improved prognosis in 5epticemia !ight glycemic controls(.mpa 5ince 2..?5 has published this study) !ransferrin 4ecombinant therapya $recotogin-alpha or Rigris or associated protein antibody

Potrebbero piacerti anche